Saturday, August 31, 2013

A gigantic book royalty check from nothing


Robert Lawrence Kuhn and John Leslie have written up a gracious and substantive reply to my recent First Things commentary on their anthology The Mystery of Existence: Why Is There Anything At All?  It will appear at the First Things website soon, as will my response.

In the meantime, a reader asks about a less serious contribution to the debate: some remarks made recently by Lawrence Krauss in a video over at Big Think.  I’ve commented on Krauss in a review of his book A Universe from Nothing for First Things and in a couple of earlier posts, here and here.  Is there anything new to be said?  Well, not by Krauss, that’s for sure.  It’s the same superficial stuff, presented with the same arrogant and uninformed confidence, and as usual barely acknowledging, much less seriously answering, the objections that have been leveled against him by atheists and theists alike.  But for that reason alone it is worthwhile exposing his errors now and again, as long as there’s a single benighted reader out there still inclined to take him seriously.

So let’s take a look.  And in good Lawrence Krauss fashion, he doesn’t hide his fallacies under a bushel but puts them on a pedestal for all to see.  This makes refutation not only easy but quick.  Consider, then, his very first sentence -- wherein, after urging us to be “careful” in our thinking he immediately flings carefulness violently to the ground and starts pummeling it.  Krauss asserts: 

[N]othing is a physical concept because it's the absence of something, and something is a physical concept. 

The trouble with this, of course, is that “something” is not a physical concept.  “Something” is what Scholastic philosophers call a transcendental, a notion that applies to every kind of being whatsoever, whether physical or non-physical -- to tables and chairs, rocks and trees, animals and people, substances and accidents, numbers, universals, and other abstract objects, souls, angels, and God.  Of course, Krauss doesn’t believe in some of these things, but that’s not to the point.  Whether or not numbers, universals, souls, angels or God actually exist, none of them would be physical if they existed.  But each would still be a “something” if it existed.  So the concept of “something” is broader than the concept “physical,” and would remain so even if it turned out that the only things that actually exist are physical.

No atheist philosopher would disagree with me about that much, because it’s really just an obvious conceptual point.  But since Krauss and his fans have an extremely tenuous grasp of philosophy -- or, indeed, of the obvious -- I suppose it is worth adding that even if it were a matter of controversy whether “something” is a physical concept, Krauss’s “argument” here would simply have begged the question against one side of that controversy, rather than refuted it.  For obviously, Krauss’s critics would not agree that “something is a physical concept.”  Hence, confidently to assert this as a premise intended to convince someone who doesn’t already agree with him is just to commit a textbook fallacy of circular reasoning.

Dutifully fulfilling his solemn pledge to give his readers “A fallacy in every sentence!”, Krauss goes on to say: 

And what we've learned over the last hundred years is that nothing is much more complicated than we would've imagined otherwise. 

So, “nothing” is complicated.  That implies that it has diverse parts, elements, aspects, or some such.  At the very least, a part or aspect A that is distinct from a part or aspect B.  But if A is different from B, then there must be something about it by virtue of which it is different.  In which case it isn’t true to say that there is nothing.  Indeed, Krauss goes on to describe “a kind of nothing” that might seem a “void” or an “infinite empty space,” when in fact “due to the laws of quantum mechanics and relativity, we now know that empty space is a boiling bubbling brew of virtual particles that are popping in and out of existence at every moment.”  Hence “nothing” is really “full of stuff.”

Well, somebody’s sure full of stuff here, but it isn’t “nothing.”  Because “stuff,” “space,” laws,” “particles,” and the like are each something.  In which case, what could it possibly mean to describe these things as aspects of “nothing”?  Have you ever heard such self-contradictory gibberish before?  Of course you have, because you’ve read Lawrence Krauss before.   

The rest is another rehash of the same brazen bait-and-switch Krauss has been repeatedly called out on by friend and foe alike for almost two years now.  Here’s how physics gives you something from nothing, where for “nothing” read “the laws of quantum mechanics,” which are, of course, not nothing but pay no attention to that sophist behind the curtain… 

Yet Krauss does think he’s got an answer to this problem.  The laws aren’t nothing, you say, but something?  Well, try this on for size: 

But even there, it turns out physics potentially has an answer because we now have good reason to believe that even the laws of physics themselves are kind of arbitrary.

There may be an infinite number of universes, and in each universe that's been created, the laws of physics are different. It's completely random. And the laws themselves come into existence when the universe comes into existence. So there's no pre-existing fundamental law. Anything that can happen, does happen. And therefore, you got no laws, no space, no time, no particles, no radiation. That's a pretty good definition of nothing. 

End quote.  What Krauss is referring to here is, of course, his preferred variation on the currently faddish “multiverse” idea, as set out in A Universe from Nothing.  But on the multiverse scenario, it is not precisely correct to say that “there’s no pre-existing fundamental law.”  By “not precisely correct” I mean “false.”  For as Krauss himself says at pp. 176-77 of A Universe from Nothing, a multiverse might exist “in the form of a landscape of universes existing in a host of extra dimensions,” or it might instead take “the form of a possibly infinitely replicating set of universes in a three-dimensional space.”  It would be governed by “the general principle that anything that is not forbidden is allowed.”  Though “we don’t currently have a fundamental theory that explains the detailed character of the landscape of a multiverse,” to make progress in such theorizing “we generally assume that certain properties, like quantum mechanics, permeate all possibilities.”  And it could turn out that there are “millions of layers” of laws.   

Needless to say, “extra dimensions,” “three-dimensional space,” “general principles,” “the detailed character of a landscape,” “properties,” “quantum mechanics,” and “millions of layers of laws” are not nothing, but a whole helluva lot of something.

Recently we had the wood floors in one of the rooms of our house redone.  Naturally we had to empty the room before work could start.  Suppose that when the wood floor guy showed up to begin, everything had been moved out except for one large bookcase.  Annoyed, he asks me why I didn’t empty the room as I had agreed to do.  Suppose I haughtily replied: 

No beds, no floor rugs, no chairs, no lamps, no bookcases.  That’s a pretty good definition of an empty room.   

My wood floor guy would no doubt reply: “No it’s not, dumbass.  You have, by your own admission, still got one bookcase in there.  Therefore it’s not empty.  I thought you taught logic?”   

Of course, the room might be close enough to “empty” for some purposes.  We might even speak loosely of there being “nothing” in it.  That’s fine for most everyday contexts, where we needn’t always use terms precisely.  But of course, it’s not good enough for every context, as the wood floor example shows.  And it certainly isn’t good enough for philosophical and scientific contexts, where we need precision.  Krauss, a prominent physicist whose work drips with contempt for the philosophers and theologians he regards as sloppy thinkers, and who urges us to be “careful” in our use of language, can’t see what the wood floor guy can.   

The reason, of course, is that the wood floor guy doesn’t have a vested interest in denying the obvious.  He hasn’t spent two years loudly shooting his mouth off about how stupid people are who think that a room with a bookcase in it isn’t really empty, and he doesn’t have a New York Times bestseller, lectures, debates, or a Big Think video devoted to confidently promoting the view that a room with a bookcase in it is empty.  Ergo he doesn’t face the utterly humiliating prospect of having to admit that since a room with a bookcase in it isn’t strictly empty, the people he’s derided as stupid actually have a point, and the book, lectures, video, etc. have all been a waste of time.   

The irony is that admitting the pickle Krauss has gotten himself into would be the one thing that might save him.  For Krauss has managed to parlay a set of completely worthless ideas into fame and fortune.  He’s gotten a big chunk of the “reality-based community” to swallow the notion that a book-length exercise in committing the fallacies of equivocation and red herring counts as Big Thinking.  He’s gotten an army of Dawkins Youth seriously to believe that while the rigorously worked out metaphysical demonstrations of an Aquinas or a Leibniz are really just loose “god of the gaps” speculations, the “multiverse” theory that is notoriously untestable and which Krauss himself admits lacks a “fundamental theory” is hard-headed empirical science.  Krauss might present his own recent career as the surest proof of his thesis: “You think something can’t come from nothing?  Just look at me! 

274 comments:

  1. Anything that can happen, does happen. And therefore, you got no laws, no space, no time, no particles, no radiation. That's a pretty good definition of nothing.

    So gods exist, in abundance? Polytheism at the very least is true, AND classical theism is still untouched?

    Interesting, Mr Krauss!

    Really, wasn't one of the central arguments of Dawkins' book (now, admittedly, very much abandoned) that God's existence was wildly improbable? Forget all the mistakes he made with his conception of God for a moment. It really does seem that if you take Dawkins and Krauss would have to concede that the 'God' Dawkins took aim at, given Krauss' multiverse speculations, exists. In fact, there's a whole lot of them.

    ReplyDelete
  2. Dr. Feser,

    While your content is always a pleasure to read, I have to say I also appreciate the effort you must go through to create the images appended to your posts.

    ReplyDelete
  3. Wiped the floor with the guy, I love it.

    ReplyDelete
  4. This poor excuse for a human being, aside from being an abject idiot, doesn't have a problem with incest

    http://subversivethinking.blogspot.com/2013/08/lawrence-krauss-it-is-not-clear-for-me.html

    ReplyDelete
  5. I wasn't keen on the multiverse idea until Stephen Barr helpfully explained what it actually is some time back: http://www.firstthings.com/blogs/firstthoughts/2013/06/04/the-large-hadron-collider-the-multiverse-and-me-and-my-friends/

    Basically, something completely different from what most people, including Krauss, think.

    ReplyDelete
  6. It's really sad that so many people are following Krauss and do not even note that he contraddicts himself.

    ---

    "There may be an infinite number of universes, and in each universe that's been created, the laws of physics are different. It's completely random. And the laws themselves come into existence when the universe comes into existence. So there's no pre-existing fundamental law. Anything that can happen, does happen. And therefore, you got no laws, no space, no time, no particles, no radiation. That's a pretty good definition of nothing"

    Indeed here Krauss is lying or maybe he's confused.

    There might be no space and time and radiation, etc... but CLEARLY there ARE some laws that govern the process of 'emergence' of these universes.

    Otherwise Krauss might as well claim that they magically appear out of nothing, and that would make no sense, scientifically.

    Moreover he seems to claim as 'fact' merely fanciful speculations.

    If really he claims that there are no laws before a universe and laws of physics are random, in the multiverse... how can you even claim something scientific about that?

    Science is about having a theory and testing it. Such theory regards 'regularities' (ie things that can be observed and reproduced in a lab) which MUST follow some 'law'.

    So whát he's saying really makes no sense from a scientific point of view.

    ReplyDelete
  7. Krauss's "nothing" seems a lot like Chesterton's Invisible Man.

    http://www.cse.dmu.ac.uk/~mward/gkc/books/innocence.txt

    ReplyDelete
  8. Why haven't you been featured on Kuhn's show yet?

    ReplyDelete
  9. I wasn't keen on the multiverse idea until Stephen Barr helpfully explained what it actually is some time back...

    I read Barr's post. He's essentially saying that since naturalism can't explain something about the Higgs particle, there must be a multiverse. "Reasoning" such as this, of course, is the same kind of scientistic jackassery that guys like Krauss are engaged in. Don't be fooled just because Barr calls himself a Catholic.

    ReplyDelete
  10. @George R

    Barr believes Francis is the true Pope.

    Do you?

    People living in Sede glass houses should not throw stone.

    Multiverse theory is compatible with Christianity.

    Thought I am skeptical about the truth of Multiverse Theory.

    ReplyDelete
  11. Multiverse theory is compatible with Christianity.

    Yachov,
    I agree, the multi-verse is definitely compatible with the New Church of Scientology, which is the version of christianity that you and Steve Barr belong to. However, it's not at all compatible with the version I belong to.

    This is because the multi-verse is nothing other than an ad hoc solution for desperate atheists (and their New-Christian-Scientologists allies) who see that it's impossible to explain certain things in our universe by natural causes. Therefore, they reason, there must be an infinite number of universes, among which even "impossible" universes like ours might be found.

    For us true Christians, on the other hand, aspects of nature that cannot be explained by natural causes "are exactly what would be predicted by our theory," as the scientists like to say. We don't need to come up with some insane, screwball explanation for it.

    Btw, your new pope's doing a great job. He's a real credit.

    ReplyDelete
  12. Ben, whatever George's issues, your comment was unnecessary. Even if there is some weird version of multiple universes, the standard scientistic version is not in the least compatible with a Christian world view, for pretty much the reason George gave. You were just pot-shotting for the fun of it.

    ReplyDelete
  13. Tony,

    Even if there is some weird version of multiple universes, the standard scientistic version is not in the least compatible with a Christian world view, for pretty much the reason George gave.

    Can you explain this view to me, in your own words? I didn't understand George's take-down of it. I did understand his take-down of the perceived reasoning behind it, and I absolutely agree the multiverse tends to get some popularity for that kind of reasoning.

    But I didn't see George arguing for the incompatibility so much.

    ReplyDelete
  14. "Yachov,
    I agree, the multi-verse is definitely compatible with the New Church of Scientology, which is the version of christianity that you and Steve Barr belong to. However, it's not at all compatible with the version I belong to."

    I don't understand this myself. Do only old Earth creationists get to heaven? Young Earth? Flat Earth? Round Earth? Big Bang deniers? Literal 6 days? What scientific theory or theological interpretation of creation should a Christian adhere to to make it to heaven George?

    ReplyDelete
  15. "However, it's not at all compatible with the version I belong to."

    Deny the authority of the Holy Father all day long, but don't you dare mention a multi-verse. That's sounds about right.

    ReplyDelete
  16. Crude, I am not suggesting that some version of "multiple universes" could not exist compatibly with Christianity. I think, though, that the standard so-called reasons posited for the usual multi-verse theories are proposed precisely because people don't want something like special Creation to be true. That is to say, the motivation behind the proposals is incompatible with Christianity, not so much the actual content of the proposals, (which all amount to sheer guesses, with hardly a fig leaf between the lot of them).

    If you look at the usual material given as "reasons", almost all of the material is of the sort like "we don't know how to explain X, so we hypothesize Y". The first problem with this is that about 2/3 of the Xs that they don't know how to explain would in fact be explained by ANY supposition of special creation, and they won't even permit that as a possible hypothesis. Like, for example, the so-called "problem" that this universe happens to be compatible with life as we know it, where as the underlying physical laws and constants could have been SO different that life as we know it could not have arisen, and it looks like the probability for OUR set up looks terribly low. That low probability, together with the fact that our existence exists, looks to them like a problem. The multi-verse allows there to be vastly many or infinite versions, so MANY universes would have very unlikely sets of laws and constants. (Never mind, for the moment, that almost to a man these same thinkers gloss over the possibility that life could arise in most, or even all other really possible arrangements of the laws and constants - they simply assume that life has to look something like life as we know it.) The "solution" is a solution to a problem that isn't really a problem: God made this universe this way for a reason.

    Another "problem" that supposedly is "solved" (it isn't) by the multi-verse is the fact that for each contingent random quantum event, nothing determines the outcome that actually comes to be, so there is nothing to explain THIS outcome instead of the other one. The supposed solution is a multi-verse in which there is a universe in which each possible outcome happens. So since all things that are possible outcomes happen, (so they say) there is no need to "explain" this specific one happening here.

    This type of reasoning is very opposed to the Christian view of contingency in God's plan for creation. The deterministic scientists view is that REAL contingency is a basic problem, the view of Christian philosophy is that it is not.

    Further, the thesis that all possible outcomes happen DOES NOT explain away the need to know why THIS one happens HERE. Even if there is some other universe where the other possible outcome happened, that does NOTHING to say why this one happened here. The scientists who posit the theory are indulging in bad science and philosophy, as well.

    ReplyDelete
  17. Tony,

    Okay. I think multiverses are BS non-science by and large, and I absolutely agree that the style of reasoning and the motivation behind them are very often crappy. It was the content I was wondering about, not the reasoning.

    For the record, I find Barr to be extraordinarily reasonable and lucid about quite a lot of science and Catholic faith. I don't think he's nearly as bad as George was suggesting, though some of that may come down to his ID interactions. And even there, he's a million times more gracious and thoughtful than, say, Ken Miller. Not to mention honest.

    ReplyDelete
  18. For all of Krauss's extreme pollyannish fanboy rhetoric about science, there's a pervasive lack of precision in his own reasoning about everything---as well as nothing.

    Jime's one hard worker over at Subversive Thinking. Just the main page right now is a riot. Probably the best and most comprehensive coverage of the entire atheist movement on the net.

    Shadows of Light is also very good, which Crude turned me on to (Thanks for that, man).

    ReplyDelete
  19. Krause is obviously pretty ignorant about philosophy; but Feser is just as ignorant about physics. Physics has more status than philosophy (for one thing, it has demonstrable practical value; for another, it is harder to do -- any moron can claim to be a philosopher; but being a physicist requires mastering a large and difficult body on knowledge.

    So I'm afraid Krause is going to keep winning, getting all those fat royalty checks and attention, while Feser sputters in obscurity. It's not fair, but neither physics nor philosophy promises to make the world fair.

    ReplyDelete
  20. Further, the thesis that all possible outcomes happen DOES NOT explain away the need to know why THIS one happens HERE

    Because you (the observer) are part of the universe. So THIS happens HERE because each individual outcome has a different version of you, each of which observes only that outcome.

    This is deeply unintuitive and rather unsatisfying, but that's how the theory works. This is how the multiverse interpretation of QM gets rid of the somewhat magic state collapse that is found in the Copenhagen interpretation. In MW, there is no collapse, just an endlessly proliferating set of possibilities, each of which when seen from within appears to have collapsed.

    ReplyDelete
  21. It'd be a more effective taunt, Anon, if you could even spell Krauss' name right. Guess he hasn't dug himself out of obscurity in some corners after all.

    Physics has more status than philosophy (for one thing, it has demonstrable practical value

    Not really. Some physics has practical value, just as some philosophy does. A good share of physics has zero practical value. Nor does it have 'more status' in the relevant sense, which is part of Krauss' problem.

    Don't fall into this sort of trap.

    any moron can claim to be a philosopher; but being a physicist requires mastering a large and difficult body on knowledge.

    Any moron can claim to be a physicist too. 'Claiming to be' and 'being' has a bit of a gulf between them.

    Rather like how Krauss claims to have solutions or at least relevant, valuable input on a variety of topics, even though in actuality he doesn't. Which is the irony - Krauss keeps whining about how people should value physics over philosophy and theology, yet philosophy and theology are where the questions important to him reside.

    To paraphrase a Krauss quote: when the subject is God and metaphysics, he has to listen to philosophers. Philosophers only barely have to listen to him.

    ReplyDelete
  22. This is deeply unintuitive and rather unsatisfying, but that's how the theory works.

    Interpretations of quantum mechanics are not theories.

    This is how the multiverse interpretation of QM gets rid of the somewhat magic state collapse that is found in the Copenhagen interpretation.

    Trading magic for magic sounds fair, but isn't exactly scientifically encouraging.

    Then again, since interpretations of quantum mechanics are philosophy rather than science, perhaps the philosophers have something of interest to say there.

    ReplyDelete
  23. >I agree, the multi-verse is definitely compatible with the New Church of Scientology, which is the version of christianity that you and Steve Barr belong to. However, it's not at all compatible with the version I belong to.

    No the last time I checked I follow the Bishop of Rome the one called Francis & I belong to the Catholic Church(same with Barr). The one the gates of Hell cannot overcome but some people sadly call themselves "Catholic" who believe it can.

    That is a far more serious error then speculations that God might have created other space time regions other than ours(which are popularly though inaccurately called other "Universes").

    >This is because the multi-verse is nothing other than an ad hoc solution for desperate atheists (and their New-Christian-Scientologists allies) who see that it's impossible to explain certain things in our universe by natural causes. Therefore, they reason, there must be an infinite number of universes, among which even "impossible" universes like ours might be found.

    Actually according to Rabbinic Tradition "God created and destroyed worlds before this one". So among those whom Our Lord said where the successors of Moses the idea God created other worlds is not incompatible.

    God can created as much as he wants whenever He wants & we cannot say boo. I don't see where Barr denies creation. At best he denies your neo-Paley pseudo Protestant view of Creation. Which he should.

    >For us true Christians, on the other hand, aspects of nature that cannot be explained by natural causes "are exactly what would be predicted by our theory," as the scientists like to say. We don't need to come up with some insane, screwball explanation for it.

    I've read my copy of Ott and Denzenger. I cannot find where it compels Christians to believe in Paley's "god"?

    >Btw, your new pope's doing a great job. He's a real credit.Btw, your new pope's doing a great job. He's a real credit.

    Yes he is isn't he! It's sad he isn't your Pope.

    ReplyDelete
  24. Seriously George cut the crap and return to the True Faith & submit to Christ's Vicar on Earth.

    "He who hears you hears me! He who rejects you rejects me & Him who sent me"-Jesus

    Enough of this High Church Protestant nonsense.

    OTOH the moral difference between a Protestant and either a Sede or Modernist so called Catholic is at least the Protestant has integrity.

    ReplyDelete
  25. @Tony.
    >Ben, whatever George's issues, your comment was unnecessary.

    As was his personal attack on Barr.

    > Even if there is some weird version of multipleuniverses, the standard scientistic version is not in the least compatible with a Christian world view, for pretty much the reason George gave.

    George hasn't given any reasons. He hasn't quoted Barr he merely name calls so I returned the favor.

    Logically given a Classic view of God & Creation I don't see how God isn't the creator of all the different "universes" in the "Multiverse"? The Atheists invoke Multiverse Theory to try to answer Fine Tuning which is the last strong Paley type of design argument left. But that has little to do with the Five Ways or Classic Theology Catholic Doctrine & Philosophy. Mind you I do find the Fine Tuning argument very compelling but I don't rely on it.

    Deny the authority of the Holy Father all day long, but don't you dare mention a multi-verse.

    Good show! BTW is there more than one Anon here?

    ReplyDelete
  26. Interpretations of quantum mechanics are not theories.

    Oh good, we're going to play "let's redefine basic words of English" again. That's always productive of insight.

    Then again, since interpretations of quantum mechanics are philosophy rather than science, perhaps the philosophers have something of interest to say there.

    You have no idea what you are talking about.

    How many philosophers do you think have usefully contributed to Everett Wheeler (or any other interpretation of QM) vs physicists?

    I suppose what you mean is that there are no empirical tests that determine which interpretation to prefer. If you think that makes it not science, then you have a schoolboy's definition of science.

    This relates to my other comment. Let's grant for the sake of argument that interpreting QM is a philosophical task rather than a scientific task. The problem is, it is still a task that requires mastery of the mathematics of QM, and very few philosophers are up to that. There are some to be sure; in fact there is a whole subfield of philosophy. But the style of philosophy based on belligerence and common sense notions (like the Aristotelianism on display here) is wholly inadequate.

    ReplyDelete
  27. @Tony

    > I think, though, that the standard so-called reasons posited for the usual multi-verse theories are proposed precisely because people don't want something like special Creation to be true.

    Special Creation? That is usually a buzz word for some species of Creationism Old Earth or Young?

    Do you have Creationist Sympathies Tony and remind me are you Catholic?

    I am only interested in Catholic dogmas of creation. Such as "God created the wold Ex-nilo(decide)."

    Big Bang Singularities don't just appear uncaused. So I don't see the problem from a Catholic perspective.

    > That is to say, the motivation behind the proposals is incompatible with Christianity, not so much the actual content of the proposals, (which all amount to sheer guesses, with hardly a fig leaf between the lot of them).

    I am not interested in "motivation" only philosophy, theology and dogma.

    The term "Big Bang" was coined as a pejorative by an Atheist Cosmologist who didn't want the Universe to have a beginning. But it is a useful term.

    ReplyDelete
  28. @Tony

    > The multi-verse allows there to be vastly many or infinite versions, so MANY universes would have very unlikely sets of laws and constants.

    I am reminded of Atheists who complain that 99.99999999999999...(add a few hundred 9's)% of our universe is hostile to Life as we know it and complain about "Why would God make all this wasted space".

    OTOH back in the 19th Century when we believed the Universe was only our Solar system of Six Planets they used to complain that in Infinite God should have made something more grand and mind blowing ly big.

    They can't make up their minds & as Crude pointed out if many universes are possible then Dawkins' "god" must exist in at least one of them.


    ReplyDelete
  29. @ Anon August 31, 2013 at 10:21 PM

    >Oh good, we're going to play "let's redefine basic words of English" again. That's always productive of insight.

    Playing "Let's bitch to cover up our basic ignorance of Philosophy & Philosophy of Science" is better how?

    >>Then again, since interpretations of quantum mechanics are philosophy rather than science, perhaps the philosophers have something of interest to say there.

    >You have no idea what you are talking about.

    And you do? Let us see.

    >How many philosophers do you think have usefully contributed to Everett Wheeler (or any other interpretation of QM) vs physicists?

    I don't know but I do know Heisenberg wrote "Philosophy of Physics". He unlike modern Physicists knew something of Plato and Aristotle and I can't imagine him making the amateurish blunders Krauss makes.

    >I suppose what you mean is that there are no empirical tests that determine which interpretation to prefer. If you think that makes it not science, then you have a schoolboy's definition of science.

    Crude is not at war with Science Only idiot Physicists who pretend that clear philosophical questions are Scientific Questions.

    >This relates to my other comment. Let's grant for the sake of argument that interpreting QM is a philosophical task rather than a scientific task. The problem is, it is still a task that requires mastery of the mathematics of QM, and very few philosophers are up to that.

    Do we need to get either The OFlynn or grodrigez (who are real mathematicians and know a good deal about Science & classic philosophy) to test this claim of yours?

    Because I don't see how Metaphysical modeling requires math? For example If I assert the philosophical view known as Materialism & do a materialist modeling of Quantum Physics I don't see where Math would be involved unless I was taking actual measurements or conducting experiments on actual subatomic events?

    A particle accelerator can't be used to prove or disprove materialism anymore than one can discuss the Atomic Weight of Natural Selection.

    Category mistake much?

    >There are some to be sure; in fact there is a whole subfield of philosophy. But the style of philosophy based on belligerence and common sense notions (like the Aristotelianism on display here) is wholly inadequate.

    Lovely another Positivist wannabe making category mistakes.

    Does anyone think it's djindra?

    ReplyDelete
  30. Where would Math be involved in a Aristotelian modeling of QM? How about Idealism? Where would the Math come in if we took a Parmedian view?

    For example if Change is real as Aristotle claimed then what purpose does Math serve in philosophically exploring that issue? If Change is not real and only stasis is real as Parmedias claimed then how is that measured by math?

    You can count what is real "Five cars in a parking lot" & what is not real "3, 2, 1, 0, -1, -2 etc)".

    Does any of this Mishegoss makes sense?

    Does anon August 31, 2013 at 10:21 PM believe modeling change (Potency being reduced to Actuality) is something we measure?

    Well maybe it will make sense after a few beers.

    ReplyDelete
  31. Oh good, we're going to play "let's redefine basic words of English" again. That's always productive of insight.

    No, it's called 'let's stick with basic words of English'.

    Interpretations of quantum mechanics are not 'theories'. They're exactly that - interpretations, metaphysical and philosophical ones. They aren't differentiated from each other by experiment - once that changes? Then we may have theories. Let me know when that changes.

    You have no idea what you are talking about.

    Right back atcha.

    How many philosophers do you think have usefully contributed to Everett Wheeler (or any other interpretation of QM) vs physicists?

    Are you under the impression that when a physicist eats a hamburger, he's 'eating a hamburger scientifically' because wow, he's a scientist, therefore everything he does is science?

    When physicists engage in philosophy, it's called philosophy. When physicists contemplate interpretations of quantum mechanics, they have not offered up scientific theories.

    I suppose what you mean is that there are no empirical tests that determine which interpretation to prefer. If you think that makes it not science, then you have a schoolboy's definition of science.

    What it makes it, my friend, is something other than a scientific theory. As any schoolboy would know. ;)

    Interpretations of quantum mechanics are based on reasoning, they make reference to science - but they are not 'science', or scientific theories. That doesn't mean they're not valuable, unless you're beholden to some nutter position that regards anything that isn't science as lacking value.

    Let's grant for the sake of argument that interpreting QM is a philosophical task rather than a scientific task. The problem is, it is still a task that requires mastery of the mathematics of QM, and very few philosophers are up to that.

    1) Mathematics isn't science either.

    2) You don't need a mastery of mathematics to engage in unhinged philosophical and metaphysical speculation about quantum mechanics. Rapt mathematical formula knowledge doesn't get you to Copenhagen's magic, or Everett's, or any others.

    3) In fact, a lack of good grounding in philosophy and metaphysics will probably hinder you every bit as much as a lack of mathematical knowledge. Arguably moreso. When a physicist with a terrible grasp of metaphysics and philosophy starts speculating about philosophical matters, you get garbage like Krauss spills.

    Better for everyone if we acknowledge when physicists aren't doing science anymore, and are off in the realms of philosophical and scientific speculation. That's the funny thing: for as much as many people talk up the superiority of science, they actually seem to dislike it. Which is why they want to stamp the 'science' label on things that manifestly are neither scientific theories, nor are scientifically evidenced.

    But the style of philosophy based on belligerence and common sense notions

    Yeah, because natural law proponents are unique in their belligerence? C'mon. And common sense notions? Common sense forms a part of any metaphysics and philosophy worth having, but it's not the sum total of any metaphysics either.

    I find quantum physics as interesting as you do, but I haven't drunk from the 'You have to disregard your common sense! What the bleep do we know?' well. Maybe some common sense notions get sacrificed, maybe they don't.

    Whatever it is, until things change... science, they ain't.

    ReplyDelete
  32. @Anonymous Coward:

    "Let's grant for the sake of argument that interpreting QM is a philosophical task rather than a scientific task. The problem is, it is still a task that requires mastery of the mathematics of QM, and very few philosophers are up to that."

    Here we go again.

    It is you who has absolutely no idea what he is talking about. The subject of QM interpretations *is* a philosophical subject. Obviously it requires a mastery of QM, along with its mathematical apparatus, and as such, it is only expected that the tiny contingent of philosophers interested in this particular subject have it. Maybe you think that philosophers should follow the stellar example of the Hawkingses and Krausses of this world and pontificate about what they know nothing of?

    The interpretation of QM is, from the physics side, inextricably entwined with the measurement problem. Apart from some scattered, largely unnoticed work, absolutely nothing was written until the 1980's (starting, say, with Ghirardi-Rimini-Weber). The elephant in the room just lay there, unacknowledged, for something like 50 years. David Albert mentions in a lecture that there is a real puzzle here, waiting some good physicist / historian to make sense of.

    "How many philosophers do you think have usefully contributed to Everett Wheeler (or any other interpretation of QM) vs physicists?"

    The latest survey of the Many-Worlds interpretation that I know of is gathered in "Many Worlds?" a collection of articles edited by Simon Saunders, Jonathan Barrett, Adrian Kent and David Wallace. Barrett (whom I had the pleasure to listen and talk with some 15 years ago) is agnostic, Saunders and Wallace defend it, and Kent rejects it. About the two defenders of MW, Saunders is a philosopher of physics and Wallace is a physicist turned philosopher. Barrett, the Agnostic, and Kent the Atheist, are *physicists*.

    It always amazes me the psychological projection of these anonymous cowards that think everyone else around here is as ignorant as they are.

    note: I reject the many worlds interpretation of QM, but David Wallace is the author of the best AI mod for the Baldur's Gate saga (SCS), so just for that, kudos to him.

    ReplyDelete
  33. Special Creation? That is usually a buzz word for some species of Creationism Old Earth or Young?

    Do you have Creationist Sympathies Tony and remind me are you Catholic?


    Ben, are you TRYING to be offensive? Have you ever seen me spouting directly creationist nonsense? Or any other nonsense, heretical or not?

    The only reason I used "special" was that after I put in Creation, I reflected back a moment on Aristotle's metaphysics leaves room for the thesis that the universe had an infinite past. And THAT kind of universe would not easily, readily quiet the physicists with their so-called "problem". Of course, Christianity (along with Judaism) would immediately present a different account of God's authorship of the universe, one in which He made it come to be at a specific moment in an identifiable state of affairs as of that moment. That moment would then bear a special relationship to all other times, unlike the infinite past universe that could be conceived with Aristotle.

    Does that sound like creationist stuff to you? Does it make using the word "special" a crypto-modernist use?

    ReplyDelete
  34. Barr doesn't accept the Many-worlds interpretation and that's not what he means by "multiverse." If you actually follow the link(s), you'll find that he's quite explicit about that.

    ReplyDelete
  35. I should probably have included a quote. From LorenzoCanuck's link, here's Barr responding to a reader's question:

    "The 'multiverse' idea is not the same as the 'many worlds' idea. The many worlds idea is one way of 'interpreting' quantum mechanics.

    The most often discussed version of the multiverse idea simply says that the universe is very large, and contains many regions in which certain numbers appearing in the laws of physics take different values.

    The 'many worlds interpretation' (MWI) of quantum mechanics says that the 'wavefunction' describing any physical system (including what’s going on right here) has countless 'branches' in which all things permitted by the laws of physics actually happen. Like you, I reject MWI."

    Before anyone wastes time arguing with Anon, it's best to check that he's gotten his facts straight in the first place.

    ReplyDelete
  36. Where would Math be involved in a Aristotelian modeling of QM? How about Idealism? Where would the Math come in if we took a Parmedian view?


    I think you are supporting my point (unwittingly, I presume).

    ReplyDelete
  37. Yes, Anon, they call it a theory. But if you read only 5 pages into this account of the "theory" you see that this is one possible alternative offering out of "dozens", and all of these rest on prior suppositions, like the "projection postulate", or on other prior assumptions about interpreting, such as that "no such additional variable are needed" - a supposition that some agree with and many do not. It absolutely clear that what is being described in the book is an HYPOTHESIS, an offered possible explanation, that might solve some difficult problems if it were tested and could be shown to hold up in testing. A single hypothesis out of dozens doesn't become a "theory" until it is tested in ways that end up shoving aside some or many of the alternative hypotheses, and maintaining the proposed hypothesis as passing the experimental tests. So, which experiments have been used to discount the many alternative hypotheses which support the MWI?

    ReplyDelete
  38. >Ben, are you TRYING to be offensive? Have you ever seen me spouting directly creationist nonsense? Or any other nonsense, heretical or not?

    I am asking you these questions because I don't remember or know. I am not accusing you of anything.

    I believe in open argument. That is people should put all their cards on the table & openly state their views.

    A simple "No I am neither a Young or Old Earth Creationist" will do & "I am or am not a Catholic".

    Believe it or not Tony I do have a life outside of comm-boxes.

    So forgive me if I don't remember who you are & your particulars. I am too self involved to remember. One of my many faults.

    I am sorry if anything I said to you was offensive.

    That was not my intent.

    ReplyDelete
  39. I> think you are supporting my point (unwittingly, I presume).

    After reading a professional opinion from the grodster I am convinced you are full of shit.

    It tragic when persons with a real professional understanding of Science & Math show up eh?

    Where is Dawkins or Krauss when you need them(not that they are much help).


    You are finished Gnu.

    ReplyDelete
  40. Hello everyone. Sorry to interrupt but I have a question about the First Way that I'm hoping the readership of this blog can answer.

    A key premise in the argument is that "only what is actual itself can actualize a given potency." Why can't an existent thing be a first mover in its own reduction from potency to act then? To be existing, after all, is to be actual.

    ReplyDelete
  41. "Why can't an existent thing be a first mover in its own reduction from potency to act then?"

    Because then there wouldn't be any reason why that reduction would take place at one time rather than another—or, equivalently, the thing would already have to be reduced to act.

    If something contained the power to reduce its own potencies to act, it wouldn't be in potency in the first place.

    ReplyDelete
  42. @New Anon

    >A key premise in the argument is that "only what is actual itself can actualize a given potency." Why can't an existent thing be a first mover in its own reduction from potency to act then? To be existing, after all, is to be actual.

    Your question is ambiguous and quite confusing. Maybe if you rephrased it?

    What is an "existing thing that is a first mover in it's own reduction from potency to act"? That doesn't make sense in Thomistic terms.


    Are you talking about "God" in some weird way? Are you talking about a self-existing creation?

    Are you really asking "Why can't a potency actualize itself"?

    Do you believe if something where self-existent it would be something that actualizes it's own potency?

    Because from an AT perspective it would make more sense for something self-existent to be Pure Act without any potency.

    For a potency to become act think something with being must impart being unless it is Being Itself.

    For something self-existent to be in essence something that actualizes it's own potency well where does it get it's initial being from?

    It's like trying to imagine a Caboose being pulled by an infinite number of unpowered Box cars.

    The more knowledgable ones here might be able to explain it better.

    Cheers then.

    ReplyDelete
  43. Why can't an existent thing be a first mover in its own reduction from potency to act then? To be existing, after all, is to be actual.

    There is nothing wrong with the thing that undergoes reduction to the actual to be an existing thing before the motion. What is impossible is that the existing thing be actual in the very respect it is going to be reduced TO actuality by the motion. A man can exist before he knows Euclid's first theorem, but he cannot know Euclid's first theorem before he comes to know Euclid's first theorem. He cannot already be actual with respect to that aspect of being that the motion is going to cause him to be actual in.

    If the man doesn't know Euclid's first theorem, his coming to be a knower in that respect has to have a cause. Now, it would be odd if the actuality "know this theorem" were to come from the man himself as first mover, since he doesn't know the theorem: If his knowledge is due to teaching, then he cannot be the teacher in the same respect he is the student.

    Of course, in some cases the student picks up the book himself and reads the theorem. But then Euclid himself is the teacher, not the student. If the student didn't need a teacher, he would COME TO KNOW the theorem without any book at all.

    In reality, even Euclid isn't the first mover of course, the chain is more involved, I was using this link as an example. It doesn't matter how many links you look at, the fact that a link is a link in a per se chain of causality for the man coming to know must originate in the only sort of mover that can move without being moved in that respect.

    ReplyDelete
  44. ---
    ---


    Catholic Church Declares War on the West

    http://occamsrazormag.wordpress.com/2013/09/01/catholic-church-declares-war-on-west/


    ---
    ---

    ReplyDelete
  45. Dr Feser,

    I'd like you to continue writing and teaching in good health and with an even temper. I'm gonna suggest you have a Martini and a deep-muscle massage before you watch Krauss twitch his way through his recent Australian 'discussions' with William Lane Craig.

    It'd be for the best.

    ReplyDelete
  46. I read the following in the First Things article:

    As Gerson says, it is no good for the atheist to say, 'Maybe there is no explanation' when the theist has just given one.

    The theist assumes theism whereas the naturalist assumes naturalism. Thus the theist’s explanation is irrelevant. When the naturalist says “maybe there is no explanation” she means “in a naturalistic reality there may not be explanations beyond a particular level, namely the level of brute facts”. I see nothing that is “no good” when a naturalist claims this.

    ReplyDelete
  47. A young physicist thinks of Feser and Krauss:

    http://letterstonature.wordpress.com/2013/09/01/feser-on-krauss/

    ReplyDelete
  48. Thanks for writing back Scott, BenYachov, and Tony.

    @BenYachov: Let me explain the reasoning behind my question a little bit. I can't address your whole comment because of space but I hope you'll see what I'm getting at in the rest of my post. From the examples that are given to illustrate the First Way, I understand 'x's reduction from potency to act' to mean a process by which x is moved from a state wherein

    x exists
    x potentially is F
    (let's call this state 1)

    to a state wherein

    x exists
    x actually is F
    (let's call this state 2).

    I want to know why there can't be some x that just moves itself from state 1 to state 2. My philosophy professor presented this scenario as an objection to the First Way.

    @Tony: My professor's scenario is impossible for a man learning Euclid's first theorem (as in your example), but still conceivable in the abstract, in spite of what you wrote. The main point of your comment, I think, is summed up in your remark that "[the man] cannot already be actual with respect to that aspect of being that the motion is going to cause him to be actual in." Well, I agree with you here, since to deny what you're saying would be absurd. However, the point you're making seems irrelevant to my professor's objection if we adopt Professor Feser's interpretation of the First Way. Professor Feser writes in Aquinas (p. 68) that "[Aquinas] is not saying that 'whatever causes something actually to be F must itself be F in some way,' but rather that 'whatever causes something must itself be actual…'" In that case, x being the cause of its own actual F-ness at state 2 hasn't been ruled out. Suppose x is the cause of its own actual F-ness; x is actual at state 1, as required, since x exists and to exist is to be actual. And x's lack of F-ness at state 1 is irrelevant to x's ability to cause itself to have actual F-ness at state 2, since whatever causes x to be actually F need not be F in any way (according to Feser's interpretation).

    Scott's reply might furnish a refutation to my philosophy professor's objection. Scott writes that if x could reduce itself from potency to act then "there wouldn't be any reason why that reduction would take place at one time rather than another…" @Scott: Why does there need to to be a reason why the reduction takes place at one time rather than another?

    I look forward to reading your responses.

    ReplyDelete
  49. "@Scott: Why does there need to to be a reason why the reduction takes place at one time rather than another?"

    Either I don't understand the question or I've answered it already. If something had the power to reduce its own potencies to actuality, then its potencies would already be actualized. Having a potency just means having to wait for something else to come along and actualize that potency.

    ReplyDelete
  50. At any rate, any elaboration missing from my own reply can be found in Tony's.

    ReplyDelete
  51. "t. When the naturalist says “maybe there is no explanation” she means “in a naturalistic reality there may not be explanations beyond a particular level, namely the level of brute facts”. I see nothing that is “no good” when a naturalist claims this."

    It is no good because no reason is given for abandoning explanation other than naturalism requires it. It is quite convenient for naturalism to do away with explanations right at the spot where any explanation would point away from naturalism. Everywhere else it is quite okay with explanation.

    ReplyDelete
  52. It is no good because no reason is given for abandoning explanation other than naturalism requires it. It is quite convenient for naturalism to do away with explanations right at the spot where any explanation would point away from naturalism. Everywhere else it is quite okay with explanation.

    I don't think you people really understand what an explanation is. Theistic "explanations", by their very nature, appear to be non-explanations to anybody not drinking the koolaid.

    I'm not sure why theists insist on offering them. Sure, in pre-modern times, gods were thought to be behind all manner of happenings and thus theistic explanations (Zeus is angry, Satan is stirring up evil, etc) were a part of everyday life. Those were the best available explanations. But we've moved on. Religion has other functions than explaining things and my advice is to get on with those and stop picking fights with science that you are inevitably going to lose.

    ReplyDelete
  53. Quantum contingency does not seem very promising for a theist world view.
    Father Jaki wrote of theologians trying to milk quantum mechanics for the free will (Miracles and Physics). But free will is rational while quantum randomness is non-rational. As CS Lewis puts it in Miracles, quantum randomness is sub-nature and the free will is supernatural.

    I believe that all cosmological speculations, including Big Bang, depend upon the non-empirical Principle of Mediocrity, which is essentially an aesthetic preference. So how much seriously should we take the cosmological speculations?.
    I did ask Prof Barr this question once in an internet discussion, but received no reply. Another objection is to apply quantum mechanics to the whole universe, that includes free willed agents.
    Quantum mechanics was originally formulated for small systems in contact with measurement apparatuses.Can it be applied to the entire universe in an unproblematic way?

    ReplyDelete
  54. The 'multiverse' idea is not the same as the 'many worlds' idea. The many worlds idea is one way of 'interpreting' quantum mechanics.

    The most often discussed version of the multiverse idea simply says that the universe is very large, and contains many regions in which certain numbers appearing in the laws of physics take different values.


    This is a good quote, Scott. This does appear to sum up Barr's notion of the "multiverse". This version does not appear to be incompatible with Christianity. So I guess I can let Steve Barr off with a warning this time.

    ReplyDelete
  55. I'm sure someone has beat me to this, but I can't help but think of this commercial anytime universes from "nothing" are discussed: http://youtu.be/YKBhXK3KApc

    ReplyDelete
  56. "Theistic 'explanations', by their very nature, appear to be non-explanations . . . "

    Perhaps to someone who doesn't understand them, which it seems clear that you don't.

    "Those were the best available explanations. But we've moved on."

    Theism "moved on" in that regard quite a long time ago. By the time of Scholasticism, it was pretty generally understood that even though theism explained in general why there was anything at all, for specific explanations it was necessary to look at secondary causes. That was well before the rise of modern science in the West and in fact was a precondition of it.

    ReplyDelete
  57. >I want to know why there can't be some x that just moves itself from state 1 to state 2. My philosophy professor presented this scenario as an objection to the First Way.

    Why does it move from state 1 to state 2? Could it be it has an essence that causes it to move from 1 to 2? So what causes it to exist here and now with that essence? OTOH "moves itself" is an ambitious use of language. A fellow I once dialoged with over at dangerous minds cited a star going supernova as an example of something "moving itself". The problem there was the fallacy of composition. A star is not really one thing it is a bunch of things. Atoms undergoing fusion, mass, gravity & of course it goes supernova if it has a very large mass and uses up the bulk of it's hydrogen during fusion & the gravity goes out of balance then BOOM.

    What you really want to say is "Why can't something be uncaused?". GEM Ascombie I believe has written extensively on it answering Hume.

    ReplyDelete
  58. Anonymous at 9/1 10:30 PM:

    When you said "stop picking fights with science," did you have some particular finding of science in mind that you think people here are disputing? If not, then what you are complaining about are philosophical views people here have relating to science. How can that not be OK here? Having such views, discussing them, criticizing them - that's the sort of thing people who read this blog do.

    ReplyDelete
  59. I always get a kick out of how defenders of naturalism - a metaphysical view notorious for brute facts and 'it just is' explanations - will project the failings of their metaphysics onto theists by talking about giving non-explanations and non-answers to problems.

    There are questions that metaphysics and philosophy address that science, by its nature, is completely incapable of addressing. Wishing really hard and pouting won't change this. Please, naturalists - stop molesting science in the name of your metaphysics-religion.

    ReplyDelete
  60. Anonymous (of September 1, 2013 at 6:59 PM),

    From the examples that are given to illustrate the First Way, I understand 'x's reduction from potency to act' to mean a process by which x is moved from a state wherein

    x exists
    x potentially is F
    (let's call this state 1)

    to a state wherein

    x exists
    x actually is F
    (let's call this state 2).

    I want to know why there can't be some x that just moves itself from state 1 to state 2. My philosophy professor presented this scenario as an objection to the First Way.


    Aquinas discusses self-moved movers in Chapter 13 of his Contra Gentiles, where he acknowledges that some part P1 of X (which part P1 is in a state of actuality) can reduce some other part P2 of X (which other part P2 is in a state of potentiality) to a state of actuality.

    Aquinas hasn't any problem with this, such as it is in and of itself. In fact, he agrees with Aristotle that animals are self-moved movers (in the sense just mentioned in the paragraph just prior).

    It seems, however, that sometimes a person will extrapolate this to the extreme, and, in doing so, think that he has discovered or crafted a viable objection to the First Way.

    But this conclusion -- that he has discovered or crafted a viable objection to the First Way -- is an erroneous conclusion.

    This erroneous conclusion is formulated or occurs as the third of three steps:

    1) reasoning that if X can reduce some part of itself which is in a state of potentiality to a state of actuality, then it is in principle possible for it to reduce, in some order and eventually, each and every part of itself which is in a state potentiality to a state of actuality;

    2) further reasoning that if x did what in principle is possible, that x then no longer would have any potentiality, and thus would be an unmoved mover; and,

    3) finally thinking, "Aha. I have (discovered or crafted) an objection to the First Way.

    But this conclusion -- or 'insight' (as a person may think or feel it is) -- in no way constitutes an objection to the First Way.

    Why not?

    Because the unmoved mover of the First Way is not merely a mover which isn't moved, but a mover which, in addition to not being moved, hasn't ever been moved.

    ReplyDelete
  61. Glenn,

    I seem to recall that Thomas makes a further argument - perhaps it is in the Summa Theologica, but I can't recall: When a person or an animal "moves itself" it is not an unmoved mover properly understood even with respect to that individual motion. This is seen with humans: when the will makes a choice, which in turn moves the muscles, which moves the body, the body is moved by the will, so the cause of the outward movement is an interior cause. But the will is moved also: it is both moved by the good apprehended, AND by God who (a) maintains the will in act as a thing that is moved by good, and (b) is the interior Mover who first moves the will toward the good.

    The body cannot have the place "at the door" that the person wants to achieve when the will moves him to go to the door, so the body cannot have the act YET to impart it to the whole. The mind has the place in intention, not in act, by knowing the place as desirable, and the will is drawn toward "being at the door" as to a good, but not determinately (not with force) because the good of being "at the door" is not apprehended as the summum bonum, and the will can incline toward a lesser good without moving thereto - it is free. That the will incline and start to adhere to the lesser good apprehended by a choice requires a mover, and the will is not THAT mover - God is. So, the human is a self-mover in a sense because he has appetites, which are inclinations toward goods, and those inclinations as interior movers move the body. But even they are not SELF-movers per se.

    ReplyDelete
  62. Anon @ Sept. 1st, 6.59:

    "x exists
    x potentially is F
    (let's call this state 1)

    to a state wherein

    x exists
    x actually is F
    (let's call this state 2).

    I want to know why there can't be some x that just moves itself from state 1 to state 2. My philosophy professor presented this scenario as an objection to the First Way."


    A potency can’t actualise itself, because if it could then by definition it wouldn’t be a potency, so when we talk of something “moving itself” from one state to another, what we actually mean (strictly speaking) is one part of that thing moving another part. But if a thing has parts, then those parts are logically and metaphysically prior to the whole thing, in which case the thing cannot be the first mover, because there’s something that comes before it. So any first mover would have to be metaphysically simple, which precludes the possession of potency.

    ReplyDelete
  63. Ed Feser, here:

    "A brain and nervous system are if anything less fundamental than the organism of which they are a part, since they are what they are only relative to the whole. "

    Mr X, above:

    "But if a thing has parts, then those parts are logically and metaphysically prior to the whole thing, in which case the thing cannot be the first mover, because there’s something that comes before it."

    Can anyone explain for me the distinction between 'less fundamental' and 'metaphysically prior'?

    ReplyDelete
  64. Tony,

    I seem to recall that Thomas makes a further argument - perhaps it is in the Summa Theologica, but I can't recall: When a person or an animal "moves itself" it is not an unmoved mover properly understood even with respect to that individual motion...

    [...]

    So, the human is a self-mover in a sense... But [humans] are not SELF-movers per se.


    Yes, indeed; thank you. I much appreciate the clarification, as well as the refinement. I'd like to say more; but I don't want to siphon attention away from the quite helpful refinement of [...] -- which, in case some reader may have missed it, is as follows:


    "This is seen with humans: when the will makes a choice, which in turn moves the muscles, which moves the body, the body is moved by the will, so the cause of the outward movement is an interior cause. But the will is moved also: it is both moved by the good apprehended, AND by God who (a) maintains the will in act as a thing that is moved by good, and (b) is the interior Mover who first moves the will toward the good.

    "The body cannot have the place 'at the door' that the person wants to achieve when the will moves him to go to the door, so the body cannot have the act YET to impart it to the whole. The mind has the place in intention, not in act, by knowing the place as desirable, and the will is drawn toward 'being at the door' as to a good, but not determinately (not with force) because the good of being 'at the door' is not apprehended as the summum bonum, and the will can incline toward a lesser good without moving thereto - it is free. That the will incline and start to adhere to the lesser good apprehended by a choice requires a mover, and the will is not THAT mover - God is."

    ReplyDelete
  65. urban jean,

    Ed Feser, here:

    "A brain and nervous system are if anything less fundamental than the organism of which they are a part, since they are what they are only relative to the whole. "

    Mr X, above:

    "But if a thing has parts, then those parts are logically and metaphysically prior to the whole thing, in which case the thing cannot be the first mover, because there’s something that comes before it."

    Can anyone explain for me the distinction between 'less fundamental' and 'metaphysically prior'?


    Uh-oh. Somebody's slip-up is showing... and it isn't either Ed Feser's or Mr X's.

    From a top-down perspecitve, the whole comes first, then the parts of which it is comprised.

    And from a bottom-up perspective, the parts come first, then that, the or any whole which the parts might comprise.

    The scenario presented by the Anonymous to which Mr X has responded had parts front and center; so, Anonymous was effectively (even if unintentionally) speaking from a bottom-up perspective.

    Mr X -- informally responding in a comm-box (unlike as Ed Feser likely would do when formally writting up an OP) -- simply responded to Anonymous while using the context of Anonymous' own directional perspective.

    That Simple.

    ReplyDelete
  66. Mr. X,

    I want to know why there can't be some x that just moves itself from state 1 to state 2.

    An actual case in point is radioactive decay. Here an atomic nucleus spontaneously moves from state 1 (say, unstable polonium-210) to state 2 (stable lead-206). Why does it move at time t1 and not at some other time t2? No reason. Nothing causes that movement to occur at time t1.

    The A-T theorist will probably insist that there must be something that causes the unstable nucleus to undergo decay at time t1. But suppose there is no such cause, as current scientific understanding has it. What’s wrong with that view? What’s wrong with the idea that it is in the nature of the nucleus of polonium-210 to spontaneously decay (with a probability of 0.5 in the next 5 days), but nothing that actually causes it to decay at the time t1 it actually does decay?. The A-T theorist will say that there is no explanation of why the nucleus decayed at time t1, and therefore that this state of affairs entails lack of explanation and thus lack of intelligibility. To which the naturalist will happily agree. So what’s wrong with the idea that reality works like that? That not only fundamental physical laws (perhaps more fundamental then the currently known), but even a vast number of elementary events that obtain every single second are unexplained, unintelligible, brute facts?

    I find there is nothing wrong. The Thomistic argument is that if one accepts a few premises (including that nature is through and through intelligible) then reason leads us to accept the existence of God. But the naturalist does not accept the premises of the argument, and it seems modern science does point to the direction that these premises are false.

    Incidentally theism does *not* entail that reality must be through and through intelligible. The existence of random (and thus uncaused) events is not prohibited by theism for the simple reason that if God wants to create a physical universe where random events do obtain then God will create such a universe. If there are such uncaused events then theism entails that there must be an explanation of why God did want to create the universe in such a way. But not that there must be an explanation for every single uncaused event.

    Actually the findings of modern science are good news for theism. Mainstream natural theology has not yet realized it, but the problem from evil (certainly the strongest argument against theism) becomes more amenable to a solution when one assumes that natural evil is ultimately random. For now the theodicist does not have to assume that there is a reason for every single instance of natural evil, but only that there is a reason why God would want to create the universe in such a way that natural evils randomly obtain. For example the much discussed argument by atheist philosopher William Rowe about the fawn dying at a forest fire becomes invalid. Thus, I say, modern science may point away from A-T metaphysics but is actually supporting theism. (But I may misunderstand A-T metaphysics, and there may be a way for it to account for random events such as the spontaneous decay of unstable nuclei.)

    ReplyDelete
  67. kuartus4,

    It is no good because no reason is given for abandoning explanation other than naturalism requires it.

    But naturalism does not require any such explanations. The naturalistic worldview entails that once the fundamental mechanical principles are reached no further explanation exists. And it entails that theistic explanations are make-believe, a kind of illusion. Actually naturalists find they can explain the mechanisms which drive theists to fall for such illusory explanations.

    ReplyDelete
  68. Anonymous 10:30 PM,

    I don't think you people really understand what an explanation is. Theistic "explanations", by their very nature, appear to be non-explanations to anybody not drinking the koolaid.

    So, what do you understand is an explanation? Do you think that only mechanistic explanations are proper explanations?

    To me it seems that any understanding which helps us make sense of our life is an explanation, especially when one finds they can consistently and fruitfully be put to work in our life. Mechanistic explanations of the kind the physical sciences use are sufficient for building a good machine, but they are insufficient for building a good life.

    Actually, by denying the validity of non-mechanistic explanations the naturalist pushes herself into a rather dark nihilistic hole. I am reminded of a quote by Richard Dawkins in his book “River out of Eden”: “In a universe of blind physical forces and genetic replication, some people are going to get hurt, other people are going to get lucky, and you won't find any rhyme or reason in it, nor any justice. The universe we observe has precisely the properties we should expect if there is, at bottom, no design, no purpose, no evil and no good, nothing but blind, pitiless indifference.

    The common naturalist simply refuses to acknowledge naturalism’s nihilistic implications and goes on to enjoy her God-created, deeply meaningful, and fundamentally good condition. Some others insist that the naturalist should herself create meaning for her life. Which of course is nonsense - it’s like saying that the naturalist should fool herself.

    Now the fact that naturalism has negative pragmatic implications does not mean that naturalism is false. It only means that, given the conceptual elegance of theism, the naturalist makes a rather silly decision to embrace such a bleak worldview. If, for discussion’s sake, we assume that naturalism and theism are equally epistemically defensible, then certainly the reasonable thing to do is to embrace theism.

    ReplyDelete
  69. Hmm..I don't suppose that one's conclusions about the ontological argument for God's existences are settled by one's grasp of the obvious. However, for my own part, I'm thinking that the ontological argument for God's existence has been debunked.

    If this is not obvious, then there are some other things that are.

    It's fairly obvious, I think, that the term 'something' is used by Kraus, as a physical concept. I don't merely infer this from the fact that he is, after all, a physicist, but from the fact that he is addressing the famous Heideggerian question 'why is there something rather than nothing'.

    ReplyDelete
  70. Glenn appears to be saying that the direction of 'metaphysical priority' between a whole and its parts depends on the polarity of our perspective. But then Mr X's conclusion, that a thing with parts cannot be the first mover, is conditional on our taking the bottom-up view. This can't be right, surely?

    ReplyDelete
  71. @ Dianelos:

    “An actual case in point is radioactive decay. Here an atomic nucleus spontaneously moves from state 1 (say, unstable polonium-210) to state 2 (stable lead-206). Why does it move at time t1 and not at some other time t2? No reason. Nothing causes that movement to occur at time t1.”

    I think there are two main problems with that line of reasoning. One is that the jump from “we don’t know why it changes at t1” to “therefore there is no cause for its changing at t1” seems to me rather unjustified. It could well be that there is a cause, but we just haven’t discovered it yet (which would hardly be an unprecedented situation in our scientific knowledge). It could also be that the change is caused by too many variables for us to measure, so whilst the decay actually happens deterministically, we can only measure it probabilistically. (Sort of like how the result of a dice roll is determined by how you throw it, the surface it lands on, etc., but we can’t measure all these variables accurately, so in practice we have to speak in terms of probability.) Since we have reason to believe that there are/have been instances where we have not yet discovered the cause of a particular phenomenon, and instances where we cannot measure variables well enough to make an accurate predictions, but no other instances where things happen without being caused, Occam’s razor if nothing else should lead us to favour one of the first two explanations rather than gratuitously positing the existence of uncaused events.

    The other major problem is that even if we accept your contention that “nothing causes that movement to occur at time t1”, it doesn’t follow that the movement itself if uncaused. If I were to build a random number generator and press the “Generate Random Number” button, there might not be a reason for it choosing one particular number over another. However, its choosing a number would not be uncaused; it would be caused by my pressing the button. So even if there is no reason for an atom to decay at t1 rather than t2, this doesn’t prove that the decay has no cause.

    “The A-T theorist will say that there is no explanation of why the nucleus decayed at time t1, and therefore that this state of affairs entails lack of explanation and thus lack of intelligibility. To which the naturalist will happily agree. So what’s wrong with the idea that reality works like that? That not only fundamental physical laws (perhaps more fundamental then the currently known), but even a vast number of elementary events that obtain every single second are unexplained, unintelligible, brute facts?”

    Well the main argument for naturalism seems to be “The physical sciences are uniquely able to increase our knowledge and understanding of the universe, philosophy and theology fail at doing this, therefore only that which can be described by the physical sciences is real!” If you then have to say “Actually, chaps, it turns out that the physical sciences can’t explain reality, it’s just a load of unexplained, unintelligible, brute facts,” this rather undercuts the rhetorical force of the first argument.

    ReplyDelete
  72. Atomic decay seems to fall under the measurement problem, which currently is not well understood.

    ReplyDelete
  73. If Aristotelians want to extend the common sense notion of cause into our understanding of the world of atoms and particles, then it seems that a fight with 20th century physics (or whatever metaphysic supports it, if you will) is inevitable, as the present discussion and an earlier one with physicist Robert Oerter would suggest. Indeed, I think one can make the case that physics abandoned the notion of cause in favour of description back in the 19th century with Maxwell's equations. The idea holds on in macroscopic physics with the Einsteinian notion that causal effects propagate no faster than the speed of light but now it seems that in quantum physics 'spooky action at a distance' is a reality. What we are to make of this I don't know. Maybe the fault lies within us and there is ineradicable indeterminacy of explanation: complete but shallow versus deep but incomplete, never deep and complete.

    ReplyDelete
  74. Nuclear decay does indeed fall under the measurement problem, as does all of quantum physics. Yet the theory makes highly accurate predictions. Given what we know now, does anyone think a resolution of the measurement problem on classical realist lines is likely?

    ReplyDelete
  75. Hi Dianelos

    You wrote earlier: “An actual case in point is radioactive decay. Here an atomic nucleus spontaneously moves from state 1 (say, unstable polonium-210) to state 2 (stable lead-206). Why does it move at time t1 and not at some other time t2? No reason. Nothing causes that movement to occur at time t1.”

    Your last two sentences are too loose. Given that you are interpreting the probabilistic findings of physics, you could more accurately write: “No reason that we know of. Nothing known to us today causes that movement to occur at time t1.” I think you recognize this yourself, and imply merely provisional knowledge, when later you write: “But suppose there is no such cause, as current scientific understanding has it.” (my emphasis)

    You then ask: “What’s wrong with that view?”

    Well, you’re on the wrong side of the principle that “there is nothing without a sufficient reason for it,” a metaphysical principle so certain that any attempt to rationally deny or even doubt it implicitly affirms it. If you’re irrationally denying or doubting it, then admitting so would be a useful timesaver.

    ReplyDelete
  76. urban jean,

    A few points:

    (1) Your comment at September 3, 2013 at 5:30 AM is interestingly inconsistent, and shows some of the problems of talking in this area without precision (which, certainly, is difficult in commments boxes). You say in three consecutive sentences (a) that talking about causes in the context of atoms and particles will require picking a fight with 20th century physics, which requires an inconsistency between the two; (b) that the case can be made that physics abandoned the concept of cause in favor of description in the nineteenth century, which requires that any talk of causes is simply in a different logical domain from anything physics is talking about; and (c) that macrophysically modern physics holds that causal effects propagate no faster than the speed of light and that quantum physics involves causation in the sense of 'action at a distance'. Each of these is a plausible thing to say, and one can certainly find physicists who will say them; and thus we have all the tell-tale signs of equivocation right here: the word 'cause' is simply not being used consistently across the board. There is no possible way to make any progress in any direction without clarifying this first; until equivocation is eliminated, all talk on the subject is hopeless.

    (2) An Aristotelian account of causation is not an "extension of the common sense notion of cause" into anything; the Aristotelian account of causation recognizes the common sense notion of cause as genuine, but it is a logical error to assume that one can therefore move in the opposite direction. Aristotle is not common sense; it just follows on Aristotelian principles that common sense is not wrong as far as it goes, not that common sense goes as far as any explanation can go.

    (3) It's a mistake to assume that realism directly implies 'classical realism', which is a specific kind of assumption about physical theories. The collapse of classical realism does not imply the collapse of the entire family of realisms. (Aristotelianism, however, does not directly imply anything on this question for this particular topic; it's just that without considering these issues, it's premature to assume that anyone has the definitive interpretation of the discoveries in question, which is necessary before we can say anything about what metaphysics they require.)

    (4) It is entirely possible that an Aristotelian account would need revision in light of findings of physics; Aristotelian principles themselves don't rule it out. This has to be established rather than simply asserted or assumed from the beginning, however, and for obvious reasons Aristotelians are reluctant to simply take appearances at face value in these matters, having a centuries-long history of being attacked on the basis of apparently contrary phenomena that turned out to be no more than apparently contrary. It is an error, however, to assume that even inconsistency with physical theory is a "fight with physics", which is an atavistic anthropomorphism; all that results is a physical puzzle, which either may be soluble with further research or may require modification of the account. Either way, it needs to be established on the specifics, not simply jumped to on the basis of one's general sense of what things imply.

    That said, it's good to raise the questions

    ReplyDelete
  77. urban jean,

    Glenn appears to be saying that the direction of 'metaphysical priority' between a whole and its parts depends on the polarity of our perspective.

    Just as to say that someone was using a certain perspective of what is isn't to say that what is is dependent on that certain perspective, so too what I in fact was saying with the words I used is not dependent on your perspective of what it was that I was saying. A bottom-up perspective is a perspective which looks at, focuses on or starts at the lower end of a hierarchy.

    Duh.

    ReplyDelete
  78. Just to repeat what others have said, because it's worth repeating...

    The A-T theorist will probably insist that there must be something that causes the unstable nucleus to undergo decay at time t1. But suppose there is no such cause, as current scientific understanding has it.

    'Current scientific understanding' has no such thing, because that understanding is not 'scientific'. It's philosophical and metaphysical, at best - and an extraordinarily weak one.

    In fact, I'll go further: there will never be a 'scientific understanding' that things occur without cause. This would be the case even if scientists were in unanimous agreement that things occur without cause, because it's not 'the opinion of scientists' that determines whether or not something is a scientific finding - it's the experiments and observations. But no one observes 'a lack of a cause', and the best experiments can convey is that a cause is not known.

    ReplyDelete
  79. "But naturalism does not require any such explanations. The naturalistic worldview entails that once the fundamental mechanical principles are reached no further explanation exists. And it entails that theistic explanations are make-believe, a kind of illusion. Actually naturalists find they can explain the mechanisms which drive theists to fall for such illusory explanations."

    I am talking about epistemic justification here. Why should we believe that there are no explanations beyond brute facts? The only reason naturalists believe such a thing is to preserve their naturalism, which is a psychological reason not a rational one. Anyone not already inclined towards naturalism has no reason to accept the existence of brute facts. What you basically said above( perhaps unknowingly) is that you don't need epistemic justification to believe in naturalism. You just believe it without any reason. Which means naturalism is unjustified and unwarranted as a belief system.

    ReplyDelete
  80. "The A-T theorist will probably insist that there must be something that causes the unstable nucleus to undergo decay at time t1. But suppose there is no such cause, as current scientific understanding has it"

    At most, current theory would say there is no physical cause(and that's debatable). But that would not rule out a metaphysical cause. And that would be the route to take if quantum physics really rules out physical causes for certain quantum phenomena since causeless events are arguably incoherent.

    ReplyDelete
  81. kuartus,

    At most, current theory would say there is no physical cause(and that's debatable). But that would not rule out a metaphysical cause.

    I think the most a current theory could say is that no physical cause has been found, and causes (x) have been ruled out. But science couldn't rule out 'metaphysical' causes.

    In fact, I'd go one better. One of the most common requests I experience in dealing with naturalists is a demand for a phenomena that cannot be explained physically. If someone holds that there are events which take place and which cannot have a physical cause - it would seem they have an answer to that question.

    ReplyDelete
  82. Crude,

    In fact I think that that was what physicist Robert Oerter was trying to say in his debate with Feser. He was saying that if current quantum theory is correct, then its not just that we have failed to find physical causes for certain quantum phenomena, but that if there were some unknown undiscovered physical cause for said phenomena that it would result in predictions which were divergent from the predictions of quantum theory. And since the predictions of quantum theory have been verified then that would signify that there are no hidden physical causes. Him being a naturalist who only accepts the reality of physical cause, this mean that causality as a general principle does not hold. But for someone not inclined towards naturalism this signifies the failure of naturalism, which is what philosopher Bruce L. Gorgon has argued in his essay " A quantum-theoretic argument against naturalism". He agrees with Oerter that quantum theory rules out physical causes. But whereas Oerter sacrifices causality because of it, Gordon sacrifices naturalism so to speak.

    ReplyDelete
  83. Crude,

    I believe that that was what Robert Oerter was trying to say in his debate with Feser. He was saying that quantum theory would be contradicted if there really were hidden undiscovered physical causes for quantum phenomena, since a theory with hidden physical causes would yield divergent predictions to the ones made by standard quantum theory. And since the predictions of standard quantum theory are the ones that are verified and borne out by observation, that signifies there arent any hidden physical causes at work behind quantum phenomena since quantum theory only gets the correct answer without them. Since Oerter regards only physical causes as real, the absence of a physical cause for quantum phenomena means the failure of causality. But to someone who is not a naturalist the absence of a physical cause of quantum phenomena means the failure of naturalism. Thats actually what the phillsopher Bruce L. Gordon argues in his essay, "A quantum-theoretic argument against naturalism". Gordon agrees with Oerter that quantum theory rules out physical causes for quantum effects, but instead of sacrificing causality as Oerter does, he sacrifices naturalism, so to speak.

    ReplyDelete
  84. kuartus,

    Gordon agrees with Oerter that quantum theory rules out physical causes for quantum effects, but instead of sacrificing causality as Oerter does, he sacrifices naturalism, so to speak.

    Interesting - I'll have to check this out to see what he has to say. If the choice is between 'no cause' and 'non-physical cause' for an event that is certainly taking place, it looks like the latter has a lot going for it. I somehow suspect that if it comes to that, all the talk of there being no possible physical cause will hastily be judged as premature.

    ReplyDelete
  85. 'I think one can make the case that physics abandoned the notion of cause in favour of description back in the 19th century with Maxwell's equations. The idea holds on in macroscopic physics with the Einsteinian notion that causal effects propagate no faster than the speed of light but now it seems that in quantum physics 'spooky action at a distance' is a reality. What we are to make of this I don't know.'

    I can offer something about this, that it's good to keep clear in one's mind, that mathematical equations are one thing, while the notion of 'cause' is something else--something else that is applied to the same thing. Let's examine the notion that 'physics abandoned the notion of cause in favour of description'. Are we clear, that according to Einstein, there is a sense in which we no longer regard gravity as a 'force'. The 'force' of gravity is Newtonian. Why does the object fall to Earth, it is pulled by gravity. Einstein gives a mathematical picture of what happens here, that no longer involves Mass. The object falling, is following a geodesic through space-time--a geodesic being the shortest distance between two points. Of course the object is moving through time, and it is moving along the geodesic.

    Similarly, the moon orbiting around the Earth, or the Earth orbiting around the sun. We see a curve, which is called 'intrinsic curvature', but the path being followed is a geodesic--the shortest distance between two points, in space-time. If you think that inertia says that an object should keep moving in a straight line, then you say 'what *causes* this deviation?' But really, in the Einsteinian model, there is no deviation.

    There is something very difficult, I think, in this, about the nature of cause. I started by saying that cause/effect, and math equations, do not interact with each other. This is, I fear, not obvious. But in a math equation, you can as easily claim that the past event is caused by the present event, as you can that the present event is caused by the past event. What is the relation between two distingishable events? This causes that, like billiard balls. Though I see much ratiocination in this thread, and technical jargon, I think the move, when it comes to cause/effect, is to be extremely suspicious from the start. It's a an abstract, and informal, concept, and Hume articulated this point long ago.

    Some have seen a crisis concerning cause/effect, which is associated specifically with Quantum physics, but I see no new crisis here, it's the same old crisis.

    Now, as to Aristotle's four causes, my comment here, is that everything has all four causes. It's not like the 'soul' is a formal-cum-final cause, as the OP asserts. Everything has, in addition to a formal and final cause, a material and an efficient cause. So, thoughts, drunken thoughts, take place in a medium of neurons soaked in alchohol. This is the material cause. And I can talk about the efficient cause, but I won't bother, because the point that everything has an efficient cause--everything, ought to be momentous enough.

    Not that you necessarily have to agree with Aristotle, but we see so invested, around here, in the far-fetched notion that Aristotle is a live option. Fine, then it's FOUR causes, people, this is four-fold, always.

    ReplyDelete
  86. Jack,

    Right, physics does not prove that there are uncaused events; it does only suggests that there are uncaused events. But that suggestion is much stronger than many realize. On the physical science alone the epistemic probability of there being uncaused events is very high – more about this in the next post.

    My main point though, as you surmised, is that I find nothing conceptually troublesome about the hypothesis that uncaused events exist. Many (naturalists and theists alike) have a strong intuition that this can’t be the case. I can understand this intuition – but I don’t share it. I find the concept of randomness very realistic, perhaps because I happen to have used it in my work. So I find nothing whatsoever counterintuitive in the idea that an unstable nucleus has the nature of *spontaneously* decaying. I have no trouble conceiving of a real random variable.

    You say that this contradicts the principle of sufficient reason (PSR). Indeed it does. So? The PSR is felt to be very strong by many, but it’s not like it’s not like it’s not controversial. Nor is its denial a show stopper: Consider a random coin, so that there is no reason for it coming up heads when one tosses it once. There is still a reason why when tossed a thousand times it will approximately come up 500 times heads. In other worlds it can very well be the case that macro-reality appears to obey PSR when in fact micro-reality doesn’t. One can easily build machines that for all practical purposes appear to be deterministic, using exclusively probabilistic building blocks. Indeed that’s exactly how michochips are built. Everything is assumed to be probabilistic, and much design work goes into removing that implicit uncertainty from the microchip’s behavior. The same goes for the design of highly efficient telecommunication systems.

    Finally, even if the PSR is true, it’s not the case that’s necessarily true. One can easily describe a logically possible world where it doesn’t hold.

    Some people conflate the PSR with the principle that “nothing comes from nothing”. I agree with the latter principle, indeed I think it’s provably true: If something comes from X, then X has the potential to produce this something and therefore X is not nothing. But the principle that nothing comes from nothing is irrelevant to our discussion. In the case of the decay of an unstable nucleus one has not “nothing” – rather one has a nucleus with a particular nature, indeed a complex nature amenable to a highly precise mathematical description.

    ReplyDelete
  87. As a matter of fact what the physical sciences do is to discover order (factual and mathematical) within physical phenomena. The realist models scientists commonly use, as well as the talk about “causality” etc are just visualization or thinking aids. Proof of that fact is quantum mechanics (QM) which was developed and has vastly grown in the last 100 years or so to the point that much of our technological civilization is based on it - *without* the description of a realist model which would produce the respective phenomenal order. Several such realist descriptions (the so-called interpretations of QM) have been suggested, are all very implausible, beyond helping people to think about QM they have no practical application whatsoever, and after 100 years of trying physicists have grown bored or disappointed about this question and have virtually stopped working on it. Nobody speaks much about this scandalous state of affairs. As physicist Nick Herbert puts it “one of the best-kept secrets of science is that physicists have lost their grip on reality”. As for philosophers of metaphysics they rarely have the required understanding of QM to produce any useful work.

    Now even though the physical sciences only discover order within phenomena and do not describe the reality which produces them, physical sciences do have important metaphysical implications in that they *delimit* the metaphysical options. So reality cannot be such as to *not* produce the phenomenal order the physical sciences reveal. Thus reality may be deterministic or indeterministic, but must be so as to produce what appear to be indeterministic phenomena. Reality may be theistic or non-theistic, but must be so as to produce physical phenomena of a very broad and very deep mathematical nature. A remarkable implication of QM is that reality is almost certainly non-local. This has falsified an intuition which naturalists thought is self evidently true (Einstein characterized non-locality as “spooky”). Indeed, non-locality fits comfortably with theism, since it is a characteristic of thought and thus fits with general providence. So, once again, we find here a case where modern science supports theism.

    Now about the issue at hand. When QM was first developed most physicists (including some of its founders such as Einstein) were deeply troubled by the fact that it appeared to say there are uncaused events. Einstein felt certain QM must be an incomplete theory and that there must be hidden causes, or “hidden variables”, which the theory did not take into account. Enters von Neumann, one of the greatest mathematicians of the 20th century, who in his Grundlagen for the first time produces the precise mathematical framework of QM. In that work he goes on to mathematically prove the following: Based on self-evidently true premises about reality, any improved scientific theory with hidden variables (i.e. variables not used by QM) would contradict observational results predicted by QM. [urban jean: that was probably physicist’s Robert Oerter’s point, which he did not describe well enough and which Ed Feser misunderstood.] Why is von Neumann’s proof so important? Because the more QM is developed and used without coming across any wrong predictions the less probable it becomes that such improved hidden-variables scientific theory exists. Today after QM’s remarkable success very few physicists expect actual errors to be discovered. For example QM contradicts Einstein’s general relativity, and everybody expects that a complete theory (the TOE) will incorporate GR’s deterministic structure in QM’s indeterministic one, and not the other way around. This far then the scientific evidence for there being physical events without physical causes is strong if not actually overwhelming.

    [continues]

    ReplyDelete
  88. To complicate matters here comes David Bohm who proposes a realist interpretation of QM which is not only based on hidden variables (indeed a mind-bogglingly vast number of fundamentally unknowable hidden variables) but is also deterministic. It’s not like von Neumann’s proof had an error; rather Bohm rejected some of the self-evident premises of the proof.

    The details of Bohm’s solution are very complicated but not crucial, for it turns out the following general principle holds: There is no proof of randomness. One can’t possibly prove the randomness of a source which is in fact random. Whatever the observational evidence, one can always produce a deterministic model that accounts for it. (Trivially: for any limited amount of data there is a mathematical function that will produce them.) Given this, any discussion about “scientific proof” is misleading, for there wouldn’t be a scientific proof even if in fact physical reality is indeterministic. Thus we are left only with evidential arguments based on the factual order which science reveals is present in physical phenomena. And, as I insist, that evidence strongly points towards physical reality being indeterministic.

    Ed Feser’s analogy with Kepler’s laws of motion is clever but in fact misleading. It is *not* the case that QM simply describes physical phenomena without recourse to physical causes. Rather the structure of QM is such that it positively suggest the *absence* of hidden physical causes.

    An interesting example in this context goes back to Einstein. He disliked so deeply the idea of uncaused events (or in general of indeterminacy) that he worked hard to prove that QM must be false. His main argument was the so-called EPR paradox. Here he described a thought-experiment where QM’s predictions entailed non-locality, which he derisively called “spooky actions at a distance”. He felt non-locality is clearly physically impossible, and therefore QM must be in error. It turned out that experimental physicists found a way to actually try out in the laboratory Einstein’s thought-experiment, found that QM’s predictions obtained, and thus that non-locality is a property of physical reality (nothing is completely conclusive, but non-locality is for all practical purposes a metaphysical fact). Moreover thinking about this experiment one sees that no hidden-variables model can account for the observational results. (The best exposition I know about this very interesting and metaphysically important case is David Mermin’s “Boojums All the Way through” – a very enjoyable read about lots of stuff beyond the ERP paradox; highly recommended.)

    In conclusion the scientific evidence strongly and positively suggests that there are physical events without hidden physical causes. The scientific naturalist is therefore quite justified in believing that such events exist. What about the theist? Given that I see no theistic defeater for the idea of uncaused events, i.e. no theistic argument the conclusion of which is that God would not create a universe where uncaused physical events occur, I am inclined to believe what the science on its face says. What’s more, as I quickly explained above, I find that the premise of there being physical events with no cause (either physical or non-physical) increases theism’s epistemic probability. Which strengthens my inclination to accept what the science on its face says, namely that uncaused events exist. (Now I understand that the existence of uncaused events contradicts a premise of A-T, but then it seems to me A-T theorists have a problem in their hands, albeit perhaps not an unsolvable one.)

    ReplyDelete
  89. Krauss was recently interviewed in Melbourne and asked about the dangers of wading into the depths of philsophy and theology. What depths? he says. Here: http://www.skandalon.net/verbatim-lawrence-krauss-on-the-depths-of-philosophy-and-theology/

    ReplyDelete
  90. Dianelos Georgoudis,

    I think that Dr. Feser defends the Scholastic notion that it doesn't make sense to talk of uncaused events well in his book Aquinas .

    In particular I think he is entirely correct that your statement that "I have no trouble conceiving of a real random variable" is extremely dubious. You cannot in fact conceive of such a thing in any meaningful sense. It is entirely beyond human understanding for there to be uncaused events.

    You fundamentally, also, do not bridge the gap between the fact that QM appears to show physical events without physical causes and there being no cause whatsover for these events. Natural science has nothing to say on non-physical causes and all the hints that its findings should simply be assumed to have a metaphysical implication (in the sense they make it likely these events are uncaused) do not bridge the gap between natural science and metaphysics.

    ReplyDelete
  91. I see no new crisis here, it's the same old crisis."

    The crisis of the extreme empiricism of Hume?

    The crisis you talk about is a modern one. The medievals and ancients, for the most part, had not confused themselves into thinking cause and effect were loose and separate.

    Dianelos you come here a lot, but you seem to lack an understanding of basic aspects of A-T.

    ReplyDelete
  92. Now, as to Aristotle's four causes, my comment here, is that everything has all four causes.

    This is false, actually; Aristotle and pretty much every major Aristotelian commentator explicitly rejects this view.

    ReplyDelete
  93. Nothing comes from nothing can't be an empirical claim, because we don't have access to regions outside the universe, and even in principle cannot observe nothing without there being an observer. So it's only going to be able to be shown to be true or false in the context of some formal system, and inevitably come down to a choice of ontologies. Also I think all of our intuitions about "becoming" our hopelessly entwined with future states of affairs arising from past states of affairs, and the experience of time, so I don't trust them when talking about states of exsistence where both of these don't hold.

    ReplyDelete
  94. Thank you, Brandon, for treating my somewhat slapdash remarks seriously. It would have been clearer had I forthrightly claimed that all of physics can be formulated without reference to cause. There is nothing in the differential equations of classical electromagnetism, say, or general relativity, that looks like a cause. In discussing the solutions to the equations one might describe what's going on in the familiar language of common sense efficient causality: moving charges cause the radiation of electromagnetic waves which cause remote charges to accelerate. The equations merely say that moving charges and radiation co-occur in certain ways. Quantum indeterminacy makes causal language quite inappropriate even in an informal presentation---particles are not understood as pieces of macroscopic matter writ very small. Quantum entanglement---Einstein's spooky action at a distance---is not understood in terms of causal effects travelling faster than light but as unexpected correlations between widely separated measurements. Upshot: physics is not formulated in causal terms at all.

    You lose me, I'm afraid, with your paragraph (2). I confess I get my Aristotle second hand. In Natural theology, natural science, and the philosophy of nature, section (4), Ed says

    The Scholastic tradition had worked out a complex and sophisticated theory of causation . . . Now the moderns gradually chucked out almost all of this nuance -- which, despite its complexity, is really just a systematic articulation of common sense thinking about causation -- as they unpacked the implications of their anti-Aristotelian revolution.

    We are left with effiicient causation which Hume then rather presciently deconstructs.

    I can't usefully add to Dianelos's excellent presentation on micro-world acausation except to offer a reference: the SEP entry on Bell's Theorem

    ReplyDelete
  95. @urban jean:

    "It would have been clearer had I forthrightly claimed that all of physics can be formulated without reference to cause."

    It would also have been more correct.

    On the other hand, it would also have deprived the claim that some quantum events are "uncaused" of any interest it ever might have had.

    ReplyDelete
  96. In particular, it appears impossible to square your statement:

    "[P]physics is not formulated in causal terms at all."

    with Dianelos's:

    "[P]hysics does not prove that there are uncaused events; it does only suggests that there are uncaused events. But that suggestion is much stronger than many realize."

    How a science that isn't "formulated in causal terms at all" could "suggest" anything whatsoever about "uncaused events" is a mystery I confess myself unable to solve; indeed it seems to leave metaphysics right where it was all along. Are you sure you're agreeing with Dianelos here?

    ReplyDelete
  97. @urban jean:

    "We are left with effiicient causation which Hume then rather presciently deconstructs."

    In what sense was this deconstruction prescient? Do you mean that he anticipated modern views?

    ReplyDelete
  98. This comment has been removed by the author.

    ReplyDelete
  99. Hi, urban jean,

    I don't think Ed's claim is correct.

    The fundamental problem with taking a positivist interpretation of physics in general is that it guarantees that physics is irrelevant to causal discussions; you can't even rule something out unless you're speaking a language that can formulate it. In general when people do appeal to physics on these matters that do assume that the equations describe causes and also that (1) causes must be deterministic, (2) there are no constituent causes, and (3) all causes are of the same kind and have the same account; Aristotelians reject all three of the latter, though.

    A minor matter, that's neither here nor there -- the efficient cause was for all practical purposes scrapped prior to Hume; a significant portion of Hume's arguments are derived directly from Malebranche, who argued that only God could possibly be an efficient cause, and thus that everything else simply operated according to contingent general laws. Hume's primary innovation was making the general laws features of the mind rather than of reality. Of course he is also the one people looked back to when reviving the view under positivism, so it's not surprising that his name comes up.

    ReplyDelete
  100. This comment has been removed by the author.

    ReplyDelete
  101. This comment has been removed by the author.

    ReplyDelete
  102. This comment has been removed by the author.

    ReplyDelete
  103. This comment has been removed by the author.

    ReplyDelete
  104. Daniel,

    This does nothing to establish your claim, however; which is that everything has all four kinds of cause. As I noted, Aristotle himself rejects this.

    ReplyDelete
  105. 'Now, as to Aristotle's four causes, my comment here, is that everything has all four causes.'

    'This is false, actually; Aristotle and pretty much every major Aristotelian commentator explicitly rejects this view.'

    This is the list? material, formal, efficient, final


    The material cause: “that out of which”, e.g., the bronze of a statue.

    The formal cause: “the form”, “the account of what-it-is-to-be”, e.g., the shape of a statue.

    The efficient cause: “the primary source of the change or rest”, e.g., the artisan, the art of bronze-casting the statue, the man who gives advice, the father of the child.

    The final cause: “the end, that for the sake of which a thing is done”, e.g., health is the end of walking, losing weight, purging, drugs, and surgical tools.

    This is fourfold, which you can see for yourself, even if, as you claim, Aristotle didn't. However, this theory is presented in its most general form in Physics II 3 and in Metaphysics V 5 (the most general presentation is repeated, in almost the same words). In both texts, Aristotle argues that a final, formal, efficient or material cause can be given in answer to a why-question. Aristotle further elaborates on causality in the rest of Physics II and in Parts of Animals I--Aristotle explores the systematic interrelations among the four modes of causality and argues for the explanatory priority of the final cause.

    Although, I have to admit, that 'pretty much every major Aristotle commentator' certainly sounds like a lively party. BYOB.

    ReplyDelete
  106. Yes, Daniel, I could cut and paste elementary expositions, too, if I wanted; the fact that Aristotle identifies four different kinds of causes does not entail that everything has all four causes, which is what you claimed. Aristotle himself clearly argues that while all causes are one of these four, not everything has all four causes.

    ReplyDelete
  107. 'Now, as to Aristotle's four causes, my comment here, is that everything has all four causes.'

    'This is false, actually; Aristotle and pretty much every major Aristotelian commentator explicitly rejects this view.'

    This is the list? material, formal, efficient, final


    The material cause: “that out of which”, e.g., the bronze of a statue.

    The formal cause: “the form”, “the account of what-it-is-to-be”, e.g., the shape of a statue.

    The efficient cause: “the primary source of the change or rest”, e.g., the artisan, the art of bronze-casting the statue, the man who gives advice, the father of the child.

    The final cause: “the end, that for the sake of which a thing is done”, e.g., health is the end of walking, losing weight, purging, drugs, and surgical tools.

    This is fourfold, which you can see for yourself, even if, as you claim, Aristotle didn't. However, this theory is presented in its most general form in Physics II 3 and in Metaphysics V 5 (the most general presentation is repeated, in almost the same words). In both texts, Aristotle argues that a final, formal, efficient or material cause can be given in answer to a why-question. Part of Aristotle’s point is that there is no one answer to this question.
    Now, upon reflection, maybe I don't want to insist on the notion that Aristotle is offering a catalog of causes, and is claiming that each thing has four different kinds of cause. At least, that's maybe not enough. Because, this account would miss that there is something ambiguous about the notion. Aristotle warns you of the ambiguity: “aition ([The Greek word ] is said in many ways.”

    An aition, as I understand it, is something that plays a role as an explanatory factor in the explanation of something. But, there are many kinds of explanations.

    Aristotle further elaborates on causality in the rest of Physics II and in Parts of Animals I--Aristotle explores the systematic interrelations among the four modes of causality and argues for the explanatory priority of the final cause.

    Example:
    1. The table is made of wood.
    2. Having four legs and a flat top makes this (count as) a table.
    3. A carpenter makes a table.
    4. Having a surface suitable for eating or writing makes this (work as) a table.

    Although, I have to admit, that 'pretty much every major Aristotle commentator' certainly sounds like a lively party. BYOB.

    ReplyDelete
  108. I suppose that you can see for yourself, though you seem used to citing others, that matter and form are not just two of the four causes, or explanatory factors,--they also are used to analyze the world statically. They tell you how it is at a given moment. But they do not tell us how it came to be that way.

    Change consists in matter taking on (or losing) form. And, efficient and final causes are used to explain why change occurs (how they came to be, and what they are for).

    Now, can the four causes be found *in nature*? As Aristotle (notoriously) held. Is there a final cause of a tree? Keeping in mind, that final causes are purposes, what a thing is for. What is the final cause of a dog, or a horse, or an oak tree? I'm inclined to let you tell me the answer that Aristotle would give--I'd like to believe your claim that 'I cut and paste elementary expositions, too', but ultimately, here's the thing, I don't believe in Aristotelian metaphysics (generally I could add 'who does?'). We're here for you. If you take the trouble, you'll argue yourself out of your own belief--I've been there.

    ReplyDelete
  109. I suppose that you can see for yourself, though you seem used to citing others,

    Says the person who has been plagiarizing others extensively. Are you really this stupid, trying to mock someone for citing (which I haven't done, incidentally; I've merely pointed out that Aristotelians don't claim what you say they do) while outright stealing someone else's words? What are you, twelve years old?

    ReplyDelete
  110. @Daniel Langlois:

    Pretty much nobody here needs elementary expositions of Aristotle's fourfold account of causation even if they were on point, and they're not.

    You said everything has all four causes; Brandon corrected you, noting (rightly) that Aristotle didn't think that and neither does pretty much anyone who has ever read him. Do you disagree?

    "[U]ltimately, here's the thing, I don't believe in Aristotelian metaphysics . . . "

    That might mean more if you raised any sort of cogent objection to it or even showed any signs of understanding it.

    "[G]enerally I could add 'who does?'"

    You may intend that as a rhetorical question, but it isn't hard to answer. Quite a few people think Aristotelian metaphysics is fundamentally sound.

    ReplyDelete
  111. @Brandon:

    "Are you really this stupid . . . ? What are you, twelve years old?"

    Seriously. Does he really think nobody else on Earth has access to the Stanford Encyclopedia of Philosophy?

    ReplyDelete
  112. "Note that Aristotle does not say that all four explanatory factors are involved in the explanation of each and every instance of natural change. Rather, he says that an adequate explanation of natural change may involve a reference to all of them. Aristotle goes on by adding a specification on his doctrine of the four causes: the form and the end often coincide, and they are formally the same as that which produces the change (Phys. 198 a 23–26)."

    Here's the relevant part of the SEP article.

    ReplyDelete
  113. Scott,

    In fairness, most people would never even know without that super-secret spy tool, Google, and its pesky dog, too.

    Anonymous,

    Thanks for pointing that out. I suppose it takes a bit of chutzpah to dismiss an interlocutor for being used to citing while simultaneously plagiarizing an encyclopedia article, which by its nature is citation-based, and which explicitly points out that the interlocutor is right. That's a pretty bold way of trying to bluff one's way through an argument in which one hasn't the faintest clue what one is talking about.

    ReplyDelete
  114. 'Are you really this stupid' and such, well, I think it would be stupid to expect any more of this charming thread.

    ReplyDelete
  115. Krauss was recently interviewed in Melbourne and asked about the dangers of wading into the depths of philsophy and theology. What depths? he says. Here: http://www.skandalon.net/verbatim-lawrence-krauss-on-the-depths-of-philosophy-and-theology/

    Sadly one doesn't even need to be truly rational to do science. And Krauss wouldn't even get it if you pointed this out to him.

    As an example of this I had a debate on facebook with a group of new atheist types, two of them (sisters) were on my friend's list due to our fellow Red Sox fandom and the fact that I knew them before their conversion to areligious fundamentalism. It started when one of the sisters threw out that tiresome new atheist meme photo about how heliocentrism "proved" that science was forever trumping "religion" (I suppose I should be grateful, I never knew that Ptolemy was a christian fundamentalist until encountering the meme). I politely pointed out that it was really an example of science's self-correction, which could be seen when Newtonian mechanics replaced heliocentrism which would itself go on to be displaced by Einstein & relativity. And the two sisters argued this with me. Worse still, they're biologists. So Krauss is right, to do science you don't need philosophy or even rationality. As he himself is living proof thereof.

    ReplyDelete
  116. 'I suppose it takes a bit of chutzpah to dismiss an interlocutor for being used to citing while simultaneously plagiarizing an encyclopedia article..'

    I wouldn't have described myself as dismissing an interlocutor for being used to citing, I believe I was suggesting that you can see things for yourself, which I think is where there is hope of finding consensus. As to 'plagiarizing' an encyclopedia thread, I wasn't counting on people not having read the same Stanford article (which I have certainly read, and more-or-less pasted from, in preference to winging it)--this is commodity stuff, basic Aristotle.

    '"Note that Aristotle does not say that all four explanatory factors are involved in the explanation of each and every instance of natural change.' In fact, two of them are, strictly speaking, never involved in the explanation of any instance of 'natural change' per se, which is a point that I have already covered. But these different causes, the four Aristotelian causes, are, to my mind, modal, alternative explanatory factors which are always questions that one may ask. A similarly useful modal scheme of explanation might be logic/math/will/*Cause*.

    However, that's it for me, I'm not concerned about whether *I* seem to be 12 years old, unless I were to persevere. I commend those who take a serious interest in these issues.

    ReplyDelete
  117. Anonymous 6:04 AM,

    In particular I think he is entirely correct that your statement that "I have no trouble conceiving of a real random variable" is extremely dubious.

    A major field in mathematics with a huge field of applicability is probability theory. Everybody who has developed it, or has learned about it, or productively uses it in her work - has no trouble whatsoever conceiving of a real random variable. I assure you of this.

    It is true that one can interpret probability theory as an idealization, and thus retain the metaphysical belief that no real random variables exist. But it is not the case that there is some cognitive barrier that keeps us from conceiving a hypothetical world in which real random sources exist. I understand that many people still have the intuition that matter is governed by classical (deterministic) mechanical rules, and that this intuition may cause them to find the idea of matter moving without any cause to push it highly distasteful. But that’s all there is to it.

    The A-T theorist may argue that such uncaused movements will be brute facts, and that brute facts are by definition unexplainable and unintelligible. That is true, but also irrelevant, since perhaps reality is not through and through intelligible as A-T assumes.

    ReplyDelete
  118. Scott,

    In particular, it appears impossible to square your statement:

    ‘[P]physics is not formulated in causal terms at all.’

    with Dianelos's:

    ‘[P]hysics does not prove that there are uncaused events; it does only suggests that there are uncaused events. But that suggestion is much stronger than many realize."


    There is no tension at all.

    Physics reveals mathematical patterns present within physical phenomena, patterns which physicists naturally enough express using mathematical equations. As urban jean says there is no referent to causality in a mathematical equation. It’s not like “3+1=4” says that adding a “1” to a “3” causes the “3” to become a “4”. Philosophers recognize this when they say that abstract objects have no causal power. Well, the entire body of scientific knowledge is abstract, and refers only and exclusively to mathematical order present within physical phenomena.

    Now the factual order which the physical sciences reveal has metaphysical implications which delimit what is metaphysically possible, since reality must be such as to produce this order. Moreover this order suggests how reality may be. Thus it *suggests* that the reality that produces this mechanical order must itself have a mechanical (aka physical) nature – in other words it suggests physical realism.

    Let me illustrate the above with an example: We know from school Newton’s great discovery that F = m * a. The order that Newton here discovers is that there is always a mathematical relation that binds three properties of a material thing, namely its mass, its acceleration, and the total force that applies to it. Newton’s formula (or the mathematical pattern Newton has discovered) says nothing about causality. It is us when conceptualizing or modelling in our minds a physical world which would produce the order described by that formula, or in general when using that formula, who project onto the formula the concept of causality. Thus we may think: I see that this ball is accelerating, therefore there must be a force causing it to accelerate. Or: If I want to cause this ball to accelerate, I should apply a force to it.

    Indeed the human condition is such that causation plays a huge role in it. Even as babies we realize that crying causes our mother to appear and to comfort us. We pass our everyday life experiencing the application of our will causing changes in phenomenal reality. Scientific knowledge greatly amplifies the power of our will to cause changes, and thus it is practical to use “causality-painted” language when using science. Or when conceptualizing what science may be saying about reality. Which is OK as long as one is aware that one is adding to the actual science. But the scientific knowledge itself is completely causation free.

    With the discovery of QM things become even more interesting, since when thinking about the structure of QM’s order people realized that it positively suggests that reality is such that uncaused events exist. It’s not just that QM like any other physical theory does not posit any causes, it makes the metaphysical assumption that such causes exist very strenuous to hold. How? By making it so that if such real causes do exist then QM must produce some false predictions, when QM is by far the most universally and most precisely confirmed scientific theory of all times.

    Since I am a theist and I believe that all of nature speaks of God’s purpose – I find this state of affairs very suggestive. It is as if God tries to tell us through science that physical nature and all its large scale order rests on microscopic non-causal events. Which has plenty of positive implications for theistic metaphysics, not least of which is that it helps us solve the problem from natural evil. For if natural evil is ultimately random then the theist does not have to look for a good reason for each instance of natural evil, but only for a good reason of why God should create the universe in such a way that instances of natural evil randomly obtain.

    ReplyDelete
  119. E.H. Munro,

    Sadly one doesn't even need to be truly rational to do science.

    True, but why sad?

    Take chess playing for example. People who play chess are rational, and a short while ago it was supposed that playing good chess was the very mark of intelligence. Today there are software programs running on cellular phones that play better chess than virtually any human. So it’s not like you need rationality for playing chess. We now know that chess playing is a mechanizable activity.

    To do good physical science one must look for stable patterns within the very complex set of physical phenomena in our environment. Of course good physicists are rational and intelligent people. But we already know that the discovery of patterns is also a mechanizable activity (it’s a field of A.I. called “pattern recognition”). I see no reason why it won’t be in the near future technically possible to implement the job of fundamental physicists (the guys who today win the Noble prices) in computers, thus proving that rationality is not needed for doing fundamental physics either. Doing physics is an extremely interesting activity, but solves the same type of problems one finds in popular magazines with quizzes.

    So where does this end? Where is there a human activity which is not mechanizable? In such cognitive abilities which entail value judgment – which I hold is the case in most of philosophy, and most obviously in ethics. In personal relations and all the creativity and self-transcending love that goes in them. And ultimately in the foundation and cultivation of the good life.

    ReplyDelete
  120. So basically QM undermines the argument from motion/change? If Aquinas had QM in his day he would not be a classical theist? Does this mean classical theism is falsified? What is left over is fideism, I guess. So at the end, Krauss is correct?

    ReplyDelete
  121. Dianelos Georgoudis,

    "the scientific knowledge itself is completely causation free."

    But the scientific knowledge is much greater than the equations. That matter consists of aggregates of atoms is statement of physics but it is not expressible as an equation.

    Thus, while the causes do not appear in the equations, it does not mean that they don't exist as items in scientific knowledge. In fact, the sciences may be defined as the study of causes.
    Physicists seek causes after all. We say that mass causes the spacetime curvature, do we not?

    ReplyDelete
  122. Dianelos:

    "With the discovery of QM things become even more interesting, since when thinking about the structure of QM’s order people realized that it positively suggests that reality is such that uncaused events exist. It’s not just that QM like any other physical theory does not posit any causes, it makes the metaphysical assumption that such causes exist very strenuous to hold. How? By making it so that if such real causes do exist then QM must produce some false predictions, when QM is by far the most universally and most precisely confirmed scientific theory of all times."

    I've heard people say this sort of thing before, but I've never been able to get a proper explanation of how this works. How exactly does QM show that if "real" (=deterministic?) causes exist, the predictions made by QM would have to be wrong?

    ReplyDelete
  123. This comment has been removed by the author.

    ReplyDelete
  124. Thanks, Brandon. I appreciate that you know a great deal more about early modern thought than I'm ever likely too. I'll have to leave the historical issues to you and Ed. What you say about Hume making general laws features of the mind chimes somewhat with what I would want to say. The obvious interpretation of the success of physics is that its equations do indeed capture the regularities we believe exist in nature. Where does this leave causation? Well, if brains are good at detecting correlations within themselves---perhaps a baby learns that the fist in its visual field is its fist because its brain detects correlations between neural activity in its visual cortex with activity in its motor cortex---then perhaps our concept of causation can be traced back to our appetite for correlation. The physicist would not deny that there are correlations in nature, even if he is sceptical of causes. However, if a sophisticated Aristotelianism can accommodate non-deterministic causes, as you suggest, then there is clearly room for an alternative account.

    ReplyDelete
  125. Bedarz says that an equation cannot express the physical fact that matter consists of aggregates of atoms. Well, what we can do with equations is to model the dynamics of an assembly of atoms in terms of motion in a 'space' of very high dimension. Each atom contributes six dimensions to this space: three for its position in real space, and three for its momentum in real space. The motion of the n real atoms is expressed as the motion of a 'point' in a 'space' of 6n dimensions. Depending on the nature of the forces between the atoms one can write an equation whose solution captures the real motions. More here

    ReplyDelete
  126. "Take chess playing for example. People who play chess are rational, and a short while ago it was supposed that playing good chess was the very mark of intelligence. Today there are software programs running on cellular phones that play better chess than virtually any human. So it’s not like you need rationality for playing chess. We now know that chess playing is a mechanizable activity."

    What if the computer's software was designed with the rule that the knight could also move diagonally like a bishop. Obviously, we would recognize the error, but the computer wouldn't. The computer would dominate humans faulty programming and all.

    So has physics been reduced to a game where men make the rules rather than trying to discover them?

    ReplyDelete
  127. Suppose that at the beginning of every per se causal chain there're really is Pure Act. Suppose also that physics became really good, so good that it could trace causal chains back to such beginnings. What would those beginnings look like to physicists?

    ReplyDelete
  128. "So has physics been reduced to a game where men make the rules rather than trying to discover them?"

    By the way, this is a sincere question.

    ReplyDelete
  129. @Dianelos:

    "Well, the entire body of scientific knowledge is abstract, and refers only and exclusively to mathematical order present within physical phenomena."

    And as at least two of us have pointed out, on this view it's impossible for physics to say anything about "causes" at all. Thus the tension with your claim that physics also "suggests" that some events are uncaused: that's a pretty amazing "suggestion" coming from a field of study that doesn't speak of causes in the first place.

    ReplyDelete
  130. @urban jean:

    "Where does this leave causation?"

    Right where it was.

    "[P]erhaps our concept of causation can be traced back to our appetite for correlation."

    (1) One of your unstated premises here is that something about modern science shows the concept of causation to be wrong (or at least reducible to something else). But according to your own view, you're not entitled to that premise. If modern physics is silent on causation, then it's silent on causation.

    (2) The concept of "causation" you're implicitly relying on here is entirely modern; Aristotelian causation isn't based on "correlations" between activities or events. So even if you were successful in arguing against the modern conception, A-T causation would be completely untouched. In fact Thomists would agree in principle with your conclusion (though not your way of arriving at it).

    ReplyDelete
  131. @urban jean:

    "However, if a sophisticated Aristotelianism can accommodate non-deterministic causes . . . "

    There's no special sophistication required; Aristotelian-Thomist causation has never been "deterministic" in the first place. But non-deterministic causation is still causation, not lack of a cause altogether.

    ReplyDelete
  132. Hi, urban jean,

    I actually think that's probably where the naturalist has to go in these matters. One of the difficulties with it is that all the obvious reasons why one would think that scientific theories have anything to do with reality are causal, because causal reasoning is the only kind of reasoning to the existence of something unknown that has been historically regarded even as reasonably accurate. I do a lot of work with the history of the problem of the external world -- how we can even know that there is anything other than our own minds -- and the only accounts that are neither obviously untenable (like solipsism) nor strongly causal are at least minimally idealist, holding that we at least don't know that anything other than minds exist. The reason is that when we switch from causation to correlation we're switching from a kind of reasoning that by its nature goes from known to unknown to something that merely deals with the coherence of things known -- and getting only so far as the coherence of our own thoughts and sensations means we don't actually ever deal with any reality other than our own minds. A second difficulty is that none of our most plausible accounts of evidence -- what makes something evidence, what counts as evidence and why, etc. -- can't be disentangled from causal reasoning. Since correlations are promiscuous we even usually judge the evidence-value of correlations by how useful or significant they are for causal reasoning. Even when scientific theories don't mention causal concepts, it's at least extraordinarily difficult to make sense of scientific experiments without causal terms, for precisely the same reason: we need a ways to distinguish genuine experiments from pseudo-experiments, for instance, but historically they've always been distinguished causally.

    Tearing out causal reasoning, in other words, is not a minor matter -- it leaves us pretty much marooned, rationally speaking, and having to build everything again literally from scratch. I do think it would be interesting to see how far one could get along such lines (Hume, as you note, did some interesting work in this direction, as did the early positivists -- but they also gave up on scientific theories being any more than useful classifications of experiences) -- it's just that it's a major project rather than a minor revision.

    ReplyDelete
  133. Just for the heck of it, I was trying to find a link to a free downloadable copy of Collingwood's "On the So-Called Idea of Causation" which was published long ago in the 'Proceedings of the Aristotelian Society'.

    It's one of those things one stumbles across in remote corners of the stacks when sleepy boredom drives you to wander from your carrel in search of some relief.

    Well no success. ... Somebody has to do something about making 70 and more year old philosophical papers more freely available on-line.

    Rant follows:

    What possible moral excuse can there be for applying - if it does apply - the Sonny Bono law to materials of this sort. (In the case of music, one can always take some satisfaction in contemplating a possible unintended effect of these laws: which would be to pull the hoarded material so far out of circulation that it becomes as socially irrelevant as any monopoly on producing Edwardian smoking jackets. The Beatles, who?)

    Anyway, the link below might be of interest. Especially to those of you who had training in the historical disciplines, and come at your philosophy somewhat from the perspective of the history of ideas.

    See section 4.5. esp.


    http://plato.stanford.edu/entries/collingwood/#OntArgColRylCor

    ReplyDelete
  134. @DNW:

    For whatever it's worth, you can read the essay for free here by opening a MYJSTOR account.

    "What possible moral excuse can there be for applying - if it does apply - the Sonny Bono law to materials of this sort."

    I'd have to check the details, but it probably doesn't apply to Collingwood's works. The Copyright Term Extension Act of 1998 (which I dislike on other grounds) applies only to materials that were still protected by copyright at the time the law was passed. Collingwood died in 1943, so offhand I expect that his works passed into the public domain according to U.S. law no later than 1993.

    (The EU has its own copyright extension law, which didn't apply only to works that were still protected when it was passed but actually revived some expired copyrights; Collingwood's were presumably among them. Even so, they'd still enter the public domain this year.)

    ReplyDelete
  135. @Dianelos:

    "A major field in mathematics with a huge field of applicability is probability theory. Everybody who has developed it, or has learned about it, or productively uses it in her work - has no trouble whatsoever conceiving of a real random variable. I assure you of this."

    You're equivocating here. You had previously written [emphases mine]: I find nothing conceptually troublesome about the hypothesis that uncaused events exist. Many (naturalists and theists alike) have a strong intuition that this can't be the case. I can understand this intuition – but I don't share it. I find the concept of randomness very realistic, perhaps because I happen to have used it in my work. So I find nothing whatsoever counterintuitive in the idea that an unstable nucleus has the nature of *spontaneously* decaying. I have no trouble conceiving of a real random variable.

    Your statement was obviously not about your ability to conceptualize random variables mathematically, and the Anon who questioned it was interpreting your meaning correctly.

    Your assurance is therefore also misplaced. There are at least two professional mathematicians who frequent this board and I have an advanced degree in the subject myself, and I daresay you won't get all of us to agree that we can conceive of a "real random variable" in the sense required for your point—which, again, was that real events could be acausal, not that mathematicians can conceive of random variables clearly enough to use them in their work.

    "It is true that one can interpret probability theory as an idealization, and thus retain the metaphysical belief that no real random variables exist. But it is not the case that there is some cognitive barrier that keeps us from conceiving a hypothetical world in which real random sources exist. I understand that many people still have the intuition that matter is governed by classical (deterministic) mechanical rules, and that this intuition may cause them to find the idea of matter moving without any cause to push it highly distasteful. But that's all there is to it."

    There's so much wrong here that I'm not even sure where to start. Suffice it to say that, properly understood, the Principle of Causality is an axiom and not an "intuition"; that your claim not to "share" that "intuition" is irrelevant unless you can demonstrate that you know what the axiom says and means, which you have not done (and what you've actually said about it strongly suggests otherwise); that it covers far more than just the activities of physical matter (and that the causation involved need not be "deterministic"); and that the repugnance of a proposition to reason involves a good deal more than "distaste."

    "Since I am a theist and I believe that all of nature speaks of God's purpose – I find this state of affairs very suggestive. It is as if God tries to tell us through science that physical nature and all its large scale order rests on microscopic non-causal events. Which has plenty of positive implications for theistic metaphysics, not least of which is that it helps us solve the problem from natural evil. For if natural evil is ultimately random then the theist does not have to look for a good reason for each instance of natural evil, but only for a good reason of why God should create the universe in such a way that instances of natural evil randomly obtain."

    Anybody can "solve the problem from natural evil" by denying that God is the cause of everything that happens. But it's an odd approach for a theist.

    Nor, of course, do all "theist[s who] believe that all of nature speaks of God's purpose" find the alleged state of affairs as suggestive as you do, so your "since" is a bit presumptuous.

    At any rate, though, the fact (if it were one) that certain quantum events were uncaused would not in any way entail the same about "natural evil."

    ReplyDelete
  136. @Dianelos:

    "People who play chess are rational, and a short while ago it was supposed that playing good chess was the very mark of intelligence. Today there are software programs running on cellular phones that play better chess than virtually any human. So it's not like you need rationality for playing chess."

    Yes, it's very much like that. You need rationality in order to create a device that has chess-playing derived intentionality. And considered purely as a physical process apart from the intentions of human programmers and observers, the activity of the chess-playing computer does not in any way constitute "playing chess."

    "Where is there a human activity which is not mechanizable?"

    Where is there a computer that can "mechanize" a human activity without first being programmed/designed to do so by a human?

    The fact that an activity can be in some way "mechanized" does not imply either (a) that the activity itself is mechanical or (b) that the artifact engaging in the "mechanized" version of the activity is doing so in any non-derived way.

    ReplyDelete
  137. Dianelos,

    So it’s not like you need rationality for playing chess.

    If the height of your ambition is simply to make moves which are not contrary to the rules of the game, you will still need at least some rationality -- how else might you soundly judge whether an intended move is legal or illegal? Taking it further, if you want to play chess, then you'll need more than merely some rationality.

    We now know that chess playing is a mechanizable activity.

    If an activity is mechanizable, then it is capable of being mechanized. And if it is capable of being mechanized, then it is not a something which is, in and of itself, mechanized. (Update: I see Scott has already addressed this. Oh well, reiterating the point in a slightly different manner won't do any harm, and it might do some good.)

    Doing physics is an extremely interesting activity, but solves the same type of problems one finds in popular magazines with quizzes.

    It is possible that you meant to say 'puzzles' rather than 'quizzes', as a quiz tests one's knowledge, while it is a puzzle which tests one's solving skills. Regardless of which word you meant to use, below is a puzzle to test your solving skills.

    You can solve the puzzle mechanistically, by using formal logic or simply through the employment of common-sense reasoning; the choice is yours. The only requirement is that you use your head, and not a computer.

    Since there are four categories each with four members, there are (4!)^2 or 331,776 possible solutions. To solve it mechanistically, all you need to do is list each of the 331,776 possible solutions, and then evaluate each one in turn to see if some conflict with the clues exists. If a possible solution conflicts with any of the clues in any way, then that possible solution isn't an actual solution. OTOH, if a possible solution doesn't conflict with any of the clues in any way, then the possible solution counts as an actual solution. But notice that it won't do to stop once you find an actual solution -- you will have to continue checking the remaining possible solutions in order to: a) make sure there aren't any other actual solutions; or, b) find all the other actual solutions which might exist.

    (cont)

    ReplyDelete


  138. Scott said...

    @DNW:

    For whatever it's worth, you can read the essay for free here by opening a MYJSTOR account."


    Thanks. Your prompting me to take a second look at that link shows it can at least be read, if not downloaded, for free. If you are willing to take the trouble to register.

    And of course anything that can be read can be downloaded; after a manner, if you are willing to go to the trouble.

    I suppose I expected a registration free PDF to be available somewhere by now.



    "I'd have to check the details, but it probably doesn't apply to Collingwood's works. The Copyright Term Extension Act of 1998 (which I dislike on other grounds) applies only to materials that were still protected by copyright at the time the law was passed. Collingwood died in 1943, so offhand I expect that his works passed into the public domain according to U.S. law no later than 1993."

    You are probably right that EU rules apply. Which would make it still protected, but not by the Mickey Mouse Law

    ReplyDelete
  139. Here is the puzzle:

    The quarterfinals of The Women's International Speed Chess Championship took place earlier this morning in Rizal Park, Manila, Philippines. The four tables at which the games were played were arrange so that each table occupied a different corner of the same square. The four games were started simultaneously, and each game involved two players -- one playing the White pieces, and one playing the Black pieces. The countries of origin for the White players were Algeria, Bermuda, Canada and Denmark, and the countries of origin for the Black players were England, Germany, Honduras and Japan. The first several moves of each game consisted of a different well-known line of opening play: Pirc Defense, Queen's Gambit Declined, Ruy Lopez, and Scotch Game. For each game, match up the country of origin for the White and Black players, the number of the table at which the game was played (1, 2, 3 or 4), as well as the opening line of play that was used.

    1. The Queen's Gambit wasn't declined at table 3.

    2. The Scotch Game was played diagonally across from where the Japanese player sat.

    3. The Japanese player and the two who played the Pirc Defense played at tables whose numbers are not consecutive.

    4. The Canadian played diagonally across from where the Ruy Lopez was played.

    5. The English player's game lasted half as long as did the game involving the player from Denmark.

    6. The German won her game, but not by playing the Scotch Game.

    7. The Algerian won her game diagonally across from where the Honduran played.

    8. The Canadian (who didn't play the Scotch Game) and the player from Denmark played at tables whose numbers are not consecutive.

    9. Table 1 was diagonally across from table 4.

    Good luck.

    ReplyDelete
  140. @Danielos Georgoudis:

    Late to the party, but here goes:

    "I find the concept of randomness very realistic, perhaps because I happen to have used it in my work."

    Randomness, a technical term of art, neither implies nor is implied by uncaused.

    "Finally, even if the PSR is true, it’s not the case that’s necessarily true. One can easily describe a logically possible world where it doesn’t hold."

    The PSR, in the Scholastic formulation, is a first-principle of being so it is metaphysically impossible for such a universe to exist.

    "But the principle that nothing comes from nothing is irrelevant to our discussion. In the case of the decay of an unstable nucleus one has not “nothing” – rather one has a nucleus with a particular nature, indeed a complex nature amenable to a highly precise mathematical description."

    The principle that nothing comes from nothing is relevant to the present discussion, for were an uncaused event to happen (and I am already slipping into Humean lingo; oh well), what it would mean is that a potency would be actualized with no actualizer doing the actualization, and this just is a denial of the principle.

    "Proof of that fact is quantum mechanics (QM) which was developed and has vastly grown in the last 100 years or so to the point that much of our technological civilization is based on it - *without* the description of a realist model which would produce the respective phenomenal order."

    This is not proof of what you think it is; rather it is proof that physics does not, and cannot, deliver the kind of conclusions you think it does.

    "Now even though the physical sciences only discover order within phenomena and do not describe the reality which produces them, physical sciences do have important metaphysical implications in that they *delimit* the metaphysical options."

    This gets things exactly backwards.

    "A remarkable implication of QM is that reality is almost certainly non-local."

    What does non-locality have to do with allegedly uncaused events?

    "This far then the scientific evidence for there being physical events without physical causes is strong if not actually overwhelming."

    There is an ambiguity lurking here, but the scientific evidence for physical events without physical causes is exactly zero. Zero as in none, zilch, puto, nada. Now maybe a causal gap sounds like a really swell idea, as it is a loophole through which God can elbow its way in, and like a puppeteer, have a share in the causal cake. It seems to me that Thomists will resist this strenuously.

    ReplyDelete
  141. Anonymous 1:38 AM

    If Aquinas had QM in his day he would not be a classical theist?

    Of course he would be a classical theist. After all I find the structure of QM supports theism. But he would have developed a metaphysics that would not be troubled by indeterminism.

    Actually I find it striking that philosophers for so many centuries did not see indeterminism coming. First they did not see its viability, i.e. that an apparently deterministic macrocosm may be grounded on an indeterministic microcosm. And second they did not see its fecundity, especially for theism (and most philosophers then were theistic).

    ReplyDelete
  142. Bedarz Iliaci,

    But the scientific knowledge is much greater than the equations. That matter consists of aggregates of atoms is statement of physics but it is not expressible as an equation.

    Actually the equations do imply that atoms under the proper conditions (e.g. of temperature) will aggregate into, say, solid chunks of matter.

    But I do agree that scientific knowledge is greater than the equations. A better example than the one you suggest would be, say, the geography of the moon. Certainly that scientific knowledge cannot be represented or captured by an equation. Some time back I think I wrote that physical science is about facts and order within physical phenomena – and it’s only the latter that may be expressed by equations.

    Perhaps the best way to put it is this: Scientific knowledge reveals stable patterns within physical phenomena. The geography of the moon is such a pattern, since every time we look at the moon we see its craters in the same configuration. Another pattern would be Newton’s F=m*a, which is present in a much larger set of physical phenomena. Fundamental physics is about the latter kind of patterns, which can be described by an equation. We could proceed and analyze which patterns are amenable to a description by an equation – but this would take us far from our current discussion. It is clear that all kinds of patterns only express an abstract relationship and do not in any way express causality. I stand by my claim that causality is something we project onto the science. (Which is quite alright – I do believe that causality exists, most obviously in the context of personal will. And scientific knowledge may help us understand better the metaphysics of causality.)

    ReplyDelete
  143. @Dianelos:

    "Of course he would be a classical theist. After all I find the structure of QM supports theism."

    But you don't find that it supports classical theism if you think it supports the view that some events are absolutely uncaused. So no, if Aquinas had believed what you believe, he would not have been a classical theist.

    "But he would have developed a metaphysics that would not be troubled by indeterminism."

    This is sheer confusion. His metaphysics isn't troubled by indeterminism anyway. Again, "indeterminism" is not the same thing as "acausality."

    ReplyDelete
  144. Mr. X

    How exactly does QM show that if "real" (=deterministic?) causes exist, the predictions made by QM would have to be wrong?

    I haven’t read von Neumann’s proof, but I think there are two special cases which illustrate how this works:

    When one reads in Mermin’s book about ERP paradox and Bell’s theorem in response to it – one plainly understands that hidden variables *cannot* account for the experimental results (in this case about correlations of observations made far from each other, and hence the only explanation is some kind of non-locality, i.e. the idea that what appears to be far in fact isn’t). Thus any reality which works by way of hidden variables would produce different observational results. Do yourself a favor and read the relevant chapter in Mermin’s book. The explanation of the experimental set-up is simple and lucidly explained (the math required is of primary school level), but the insight about the relation between physics and metaphysics is momentous. Finally, please observe that in the context of our discussion one does not need a general proof; suffice to have one case where hidden variables cannot account for the actual result.

    The second argument is based on what may or may not be Robert Oerter’s idea. Let’s suppose there is a hidden parameter which causes a polonium-210 nucleus to decay at a particular time. We know that polonium-210 has a half-life of 140 days, i.e. there is 50% probability of a polonium-210 nucleus to decay within 140 days. If there is a physical parameter which causes each nucleus to decay at a particular point in time then (at least in principle) we could use it to filter the fast from the slow decaying nuclei. Thus we could take a chunk of polonium-210 and separate it into two halves – one containing all the nuclei which will decay within 140 days and one containing all the nuclei which will decay later. All nuclei in the first chunk of polonium-210 will then decay within 140 days. But this contradicts QM’s prediction that only 50% of the nuclei within *any* chunk of polonium-210 will decay within 140 days.

    ReplyDelete
  145. Anonymous 6:13 AM

    So has physics been reduced to a game where men make the rules rather than trying to discover them?

    There is difference between chess playing and fundamental physics research. The former is indeed based on human-made rules. Not so the latter. Fundamental physics research consists in discovering deep patterns present within physical phenomena. And this activity I say is mechanizable. There are already computerized systems that search for deep patterns (e.g. in the context of the search for underground oil fields). Fundamental physics research is a much harder case of pattern recognition – but , again, I don’t see why it can’t be computerized. There are few if any problems about quantities which cannot in principle be computerized.

    ReplyDelete
  146. @Dianelos:

    "If there is a physical parameter which causes each nucleus to decay at a particular point in time then (at least in principle) we could use it to filter the fast from the slow decaying nuclei."

    . . . assuming, among other things, that the filtration process itself didn't affect that parameter.

    "But this contradicts QM's prediction that only 50% of the nuclei within *any* chunk of polonium-210 will decay within 140 days."

    Aren't you walking into a sorites paradox here? Which half of one polonium-210 nucleus will decay within 140 days?

    ReplyDelete
  147. Scott,

    that's a pretty amazing "suggestion" coming from a field of study that doesn't speak of causes in the first place.

    Again, take Newton’s equation F=m*a. It only expresses a mathematical relationship between three physical properties, correct? So it does not say anything about causality. Nevertheless when we see a ball accelerating we figure that there must be a force *causing* that acceleration. This thought is not entailed in Newton’s equation but is suggested by it. It’s a common or natural way to interpret what Newton’s equation says about the real world. But as is the case of all such metaphysical interpretations it may also be wrong.

    Apart from such suggestions, physical science has specific metaphysical implications, namely that reality cannot be such as to *not* produce observations with the order science has revealed. And that order in the case of QM is such that it strongly suggests there are no physical causes for many physical changes. Actually if QM is true then it is proven that such physical causes do not exist, for the existence of such physical causes would render QM false.

    ReplyDelete
  148. @Dianelos:

    "And this activity I say is mechanizable."

    So you do. But unless you say it's mechanical (or that the "mechanized" activity continues to have any non-derived intentionality even if all human beings disappear from the cosmos), it's not clear why that matters.

    ReplyDelete
  149. Brandon,

    Tearing out causal reasoning [snip]

    There is no reason to tear down causal reasoning. It’s often a useful thinking aid, as long as one is not mislead into believing that what is found helpful when thinking must also be true. Here’s an example. One realist interpretation of QM is the so-called many worlds interpretation. I personally find that interpretation completely implausible, not only because it contradicts theism, but also because it has mind-bogglingly weird metaphysical implications. At the same time I find it is useful for thinking about QM. Thus I conceptualize a hypothetical world where the many-worlds interpretation holds, and in which quantum observations are identical to ours.

    ReplyDelete
  150. Scott,

    I have an advanced degree in the subject myself, and I daresay you won't get all of us to agree that we can conceive of a "real random variable" in the sense required for your point—which, again, was that real events could be acausal, not that mathematicians can conceive of random variables clearly enough to use them in their work.

    Strictly speaking, what’s required for my point is only that one can conceive of events in a hypothetical world to be acausal. And I can’t imagine a mathematician working with probability theory being unable to conceive of a hypothetical world where random variables exist. But if one can conceive of such a world, then one must concede that it is logically possible that in our world random variables exist also. One’s metaphysical beliefs may render such existence metaphysically impossible, but on pain of begging the question one may not posit one’s metaphysical beliefs as given.

    the Principle of Causality is an axiom and not an "intuition"

    Well, to cut to the chase, if you ask any physicist if there is physical cause for a unstable nucleus to decay at time t1 and not at time t2 – she will probably answer you that physics does not mention or require any such cause and that for all we know there isn’t one. Indeed that she would be surprised if some such physical cause were ever discovered.

    that it covers far more than just the activities of physical matter

    Perhaps you’d like to argue with the abovementioned physicist that since the axiom of the Principle of Causality applies universally, if she believes there is no physical cause then the axiom requires her to believe that there is a non-physical cause.

    Anybody can "solve the problem from natural evil" by denying that God is the cause of everything that happens.

    Ah, but the hypothesis here is that God did cause creation to be such that instances of natural evil randomly obtain. And it’s not trivial to explain why would want this. But at least the problem has now a much more concise form, than having to answer why God should cause every single instance of natural evil.

    You need rationality in order to create a device that has chess-playing derived intentionality. And considered purely as a physical process apart from the intentions of human programmers and observers, the activity of the chess-playing computer does not in any way constitute "playing chess."

    Right. So? The point remains that a machine which successfully beats human opponents in games of chess is not rational. Similarly a machine which produces cutting edge discoveries in fundamental physics is not rational either.

    Where is there a computer that can "mechanize" a human activity without first being programmed/designed to do so by a human?

    Actually one can build computers which program themselves. Still I don’t see your point. Surely you are not claiming that computers are rational, or that they can do all we do?

    ReplyDelete
  151. @Dianelos:

    "Again, take Newton's equation F=m*a. It only expresses a mathematical relationship between three physical properties, correct? So it does not say anything about causality. Nevertheless when we see a ball accelerating we figure that there must be a force *causing* that acceleration. This thought is not entailed in Newton's equation but is suggested by it."

    No, it isn't. It's suggested by the metaphysical preconceptions of the person interpreting the equation, which by your own account says nothing about causality.

    "Apart from such suggestions, physical science has specific metaphysical implications, namely that reality cannot be such as to *not* produce observations with the order science has revealed."

    Interesting. How does it imply this without saying anything about causality?

    ReplyDelete
  152. @Dianelos:

    "Well, to cut to the chase, if you ask any physicist if there is physical cause for a unstable nucleus to decay at time t1 and not at time t2 – she will probably answer you that physics does not mention or require any such cause and that for all we know there isn't one."

    If this was "the chase," you might have saved time and trouble by "cut[ting] to" it a long time ago. That a physicist, speaking as a physicist, might say that physics doesn't mention or require such a physical cause is admitted on all sides. That's pretty much anon-starter, though, for reasons quite a few of us have already canvassed repeatedly in this thread and others.

    ReplyDelete
  153. @Dianelos:

    "[C]ausal reasoning [is] often a useful thinking aid, as long as one is not [misled] into believing that what is found helpful when thinking must also be true. . .  [For example] I find [MWI] is useful for thinking about QM."

    But you apparently don't find it true. And it is truth, not "usefuless," that's at issue here.

    Well, I'll bow out for now; I feel like I'm shooting skeet.

    ReplyDelete
  154. @grodrigues,

    "There is an ambiguity lurking here, but the scientific evidence for physical events without physical causes is exactly zero. Zero as in none, zilch, puto, nada. Now maybe a causal gap sounds like a really swell idea, as it is a loophole through which God can elbow its way in, and like a puppeteer, have a share in the causal cake. It seems to me that Thomists will resist this strenuously."

    In his paper on inertia and the argument from motion, Feser suggested that a viable explanation for inertial motion would be the action of angels. If that was the case then it would follow that at least some behaviour of some natural substances didn't flow from their essence. It would be no different than the case of quantum phenomena if indeed quantum effects didn't result from the powers of natural substances. If Aristotelian metaphysics isn't opposed to the first scenario, why would it be opposed to the second?

    ReplyDelete
  155. @kuartus:

    You're asking grodrigues, not me, but I'll reply anyway.

    "If Aristotelian metaphysics isn't opposed to the first scenario, why would it be opposed to the second?"

    In principle it isn't; if it turned out on metaphysical grounds that certain events (quantum or otherwise) depended on supernatural causes, it would just acknowledge as much and move on. The issue here is that there's nothing in physics that can tell Aristotelian(-Thomist) metaphysics that that's the case.

    Moreover, there's nothing in physics that could possibly even "suggest" (let alone positively show) that a physical event lacks any cause whatsoever—a claim that would rule out angels (and God) as surely as it would rule out physical causes.

    ReplyDelete
  156. @Dianelos:

    One more point.

    "Ah, but the hypothesis here is that God did cause creation to be such that instances of natural evil randomly obtain."

    Ah, but the hypothesis here is that God caused creation to include certain events that have no causes, including divine ones—in other words, that God made it happen that there are events that God didn't make happen.

    ReplyDelete
  157. @Scott,

    " Moreover, there's nothing in physics that could possibly even "suggest" (let alone positively show) that a physical event lacks any cause whatsoever—a claim that would rule out angels (and God) as surely as it would rule out physical causes."

    I agree, but its been argued( by both theists and atheists) that physics has ruled out physical causes for certain quantum phenomena. Its only for the naturalist that the absence of a physical cause means the absence of any cause since cause=physical cause.

    ReplyDelete
  158. @kuartus:

    "[It's] only for the naturalist that the absence of a physical cause means the absence of any cause since cause=physical cause."

    And if that's not what the supposed non-naturalist means even though his position seems to imply it, then it's up to him to clarify his point. That an event may lack a purely physical cause just isn't very interesting from an A-T point of view, and it isn't in any way the counterexample to the Principle of Sufficient Reason (or Principle of Causality) that it's made out to be.

    ReplyDelete
  159. Scott says that physics is no guide to our understanding of 'causation'. I beg to differ. Physics is in the business of offering explanations of certain natural phenomena. Suppose we heat some gas in a closed container. Its pressure rises. Why? The explanation involves an understanding of what heat and pressure are in terms of the motions of the particles we understand constitute the gas. How does this fit with the Aristotelian taxonomy of explanation? I don't think it fits at all. It's not that the Aristotelian scheme is wrong. It's more that it is irrelevant or even inapplicable. It's troubling that a system that lays claim to universality falls short in this regard. But neither does the explanation fall back on the common sense notion of 'cause' that some say is a spavined descendent of the Aristotelian efficient cause. True, my remarks to date have been directed at this modern conception of cause. Everywhere we are offered 'causal' explanations of natural phenomena I believe physics offers a better explanation that makes no reference to causes. I won't say causes can be reduced because in the end I doubt they are there in any fundamental sense. Further, in so far as the germ of the modern concept lies within the Aristotelian efficient cause then to that extent reality eludes the earlier concept too. But my main point is that the form of explanation we find in physics and the natural sciences generally lies outside the Aristotelian framework.

    ReplyDelete
  160. @urban jean:

    "Scott says that physics is no guide to our understanding of 'causation'. I beg to differ."

    Then you're differing with yourself, as you've already made quite a lot of hay out of the fact/claim that physics can be entirely formulated without reference to causation.

    "Physics is in the business of offering explanations of certain natural phenomena."

    It is? What happened to your earlier view that "physics abandoned the notion of cause in favour of description back in the 19th century"? Description isn't explanation.

    "Suppose we heat some gas in a closed container. Its pressure rises. Why? The explanation involves an understanding of what heat and pressure are in terms of the motions of the particles we understand constitute the gas. How does this fit with the Aristotelian taxonomy of explanation? I don't think it fits at all."

    I'm more than a little bemused by your claim that an explanation in terms of "motion" doesn't "fit[] at all" with Aristotelian metaphysics.

    ReplyDelete
  161. Danielos,

    There is no reason to tear down causal reasoning. It’s often a useful thinking aid, as long as one is not mislead into believing that what is found helpful when thinking must also be true.

    This is irrelevant to the point; heuristic functions are in themselves neither explanatory nor justifying.

    ReplyDelete
  162. Slow down a bit Scott. That's just the opening sentence! The rest of the piece elaborates on why I think the kind of explanation we find in physics extends causation understood as the Aristotelian taxonomy of explanations.

    Do you think that the kinetic theory, which is largely descriptive of what gases are, has no explanatory value?

    If you do think the kinetic theory explanatory then which of the four Aristotelian kinds of explanation does it fall under do you think?

    ReplyDelete
  163. @urban jean:

    "Slow down a bit Scott. That's just the opening sentence!"

    I dealt with what followed in the rest of my post.

    "If you do think the kinetic theory explanatory then which of the four Aristotelian kinds of explanation does it fall under do you think?"

    All four of them in one way or another, but primarily efficient causation. A gas doesn't qualify as a "substance" according to A-T metaphysics; all of its properties reduce to the (local) motion of its constituent particles, and the potential for motion of any one particle is actualized by the other particles as efficient causes.

    Of course this brief reply isn't intended to be complete, and others may add to it or correct it.

    ReplyDelete
  164. Dr Feser, did you see this article against epiphenomenalism? It's actually quite a clever argument:

    http://voices.yahoo.com/conscious-illusion-consciousness-12294896.html?cat=58

    ReplyDelete
  165. "Scientific knowledge reveals stable patterns within physical phenomena."

    To be precise, the stable patterns i.e. the correlations that we discern in phenomena are the raw material for the Theory that is the object of a scientific investigation.

    The scientist seeks to understand the natural phenomena i.e. to know the causes.

    Causation is entirely different from correlation. A machine can do correlation. As CS Lewis wrote somewhere, expectation (i.e behavior based upon correlation) is characteristic of animal behavior, not of rational behavior.

    That sun rises everyday in east, and so we expect it to rise tomorrow, again in east, is an expectation.

    But, to know why sun rises in the east (because the earth itself is rotating on its axis) and thus to know that rise will rise in east tomorrow, if nothing else intervenes, this is a scientific understanding.

    That is, the intellect perceives causes, either through discursive or intuitive reason.
    Correlations do not require reason and are used by animals as well. Thus contrary to many atheist in particular, all rationality can not be reduced to Bayes' Theorem.

    ReplyDelete
  166. >>So I'm afraid Krause is going to keep winning, getting all those fat royalty checks and attention, while Feser sputters in obscurity. It's not fair, but neither physics nor philosophy promises to make the world fair.

    Why do people, like anon, resort to "Well, he's famous and wealthy and can have any woman he wants. You're not, so you shouldn't say anything negative." It's like pulling the race card. It's a sad attempt to end the discussion.

    ReplyDelete
  167. I didn't mean to, but you nerds have made me read through pretty well all these comments. I just couldn't stop. Thanks, guys.

    ReplyDelete
  168. --
    --

    Darwinism & Final Causation

    http://occamsrazormag.wordpress.com/2013/09/06/darwinism-final-causation-iii/

    --
    --

    ReplyDelete
  169. Scott,

    But you don't find that it supports classical theism if you think it supports the view that some events are absolutely uncaused.

    By “classical theism” I characterize all theistic metaphysical theories which are consistent with the core commitments entailed by St. Anselm’s definition, the Gospels, the Nicene Creed, and perhaps the first Fathers. I assume that A-T metaphysics is consistent with these core commitments, but it’s not of course the case that only A-T metaphysics can be considered to be thus consistent. And therefore it’s not the case that only A-T metaphysics may be called “classical theism”. Let’s have some humility here.

    And in any case we are after truth, and any debate about what can properly be called “classical theism” is a matter of convention and quite immaterial.

    "indeterminism" is not the same thing as "acausality."

    Sure, “indeterminism” a much broader concept. But in any indeterministic system there are acausal facts. Here’s the proof: By definition if a system is deterministic then some state in it can evolve into at least two future states. But then nothing in the system causes it to evolve into one of these potential states rather than the other. For if there were some cause in the system that necessitates its evolution into one and not the other future state, then it would not be true that it can potentially evolve into different states.

    assuming, among other things, that the filtration process itself didn't affect that parameter.

    Right, good point. That’s why I put the “at least in principle” qualifier. Please observe that I was only trying to answer Mr. X’s question of how it might be the case that the structure of a physical theory implies that either no hidden physical causes exist or else that some of its predictions is false.

    But unless you say it's mechanical (or that the "mechanized" activity continues to have any non-derived intentionality even if all human beings disappear from the cosmos), it's not clear why that matters.

    The discussion was not about intentionality per se, but about rationality. We were discussing whether rationality is needed for producing novel scientific knowledge. I argued that it is not needed pointing out that a machine blindly executing a program could do it. I assume we agree that a machine will have derived intentionality but not rationality.

    In this context here’s a puzzle: I once wrote a software program which wrote another program. Testing the latter program I found it worked very well. I then tried to understand why it worked so well, but couldn’t understand a thing. Now does that latter program have derived intentionality or not? I assume it must, but then where does that intentionality derive from?

    No, it isn't. It's suggested by the metaphysical preconceptions of the person interpreting the equation, which by your own account says nothing about causality.

    The thought “there must be a force causing the ball to accelerate” is certainly suggested by Newton’s F=m*a. For if one does not know of this formula then one will not produce that thought. But I agree that the thought is only suggested if one’s metaphysical preconceptions entail the existence of causality. Which in our case is a universal preconception by the age we are three.

    [continues]

    ReplyDelete
  170. I claimed that “Physical science has specific metaphysical implications, namely that reality cannot be such as to *not* produce observations with the order that science has revealed.

    To which you comment: “How does it imply this without saying anything about causality?

    I meant the above bit epistemically – hence my use of “implication”. Anyway, let me rephrase the idea thus: The order within physical phenomena revealed by the physical sciences delimits what we may reasonably believe about how reality is. Namely we may not reasonably believe that reality is such as to not produce that order. Thus scientific knowledge does have concrete metaphysical implications, albeit of a negative kind.

    That a physicist, speaking as a physicist, might say that physics doesn't mention or require such a physical cause is admitted on all sides.

    But then the fact that reasonable people produce useful work about physical movement without assuming either physical or non-physical causes – proves that it is not the case that reason prohibits us from conceiving movement without any cause. Physicists pass much of their productive intellectual life conceptualizing a reality where much moves without any cause whatsoever. Which shows, I argue, that one of the main premises of A-T is not as basic or as necessary for intelligibility as A-T theorists imagine.

    And it is truth, not "usefuless," that's at issue here.

    Exactly. But then we should not assume that what is found to be useful to the intellect necessarily leads to truth, especially to metaphysical truth.

    in other words, that God made it happen that there are events that God didn't make happen.

    Right, even though a better wording would be that God willed a world in which some events not willed by God obtain. So, what do you think is unacceptable with that hypothesis? Surely if God wants to create and give to us a random die, i.e. a die that will produce a random result every time we throw it – it’s not like God can’t do it.

    ReplyDelete
  171. Glenn,

    If an activity is mechanizable, then it is capable of being mechanized. And if it is capable of being mechanized, then it is not a something which is, in and of itself, mechanized.

    Perhaps so, but where did I claim the opposite? I was only arguing that the (presumably designed by humans) machine which produces novel scientific results should not be considered rational.

    It is possible that you meant to say 'puzzles' rather than 'quizzes'

    Right, that would be the better word choice.

    ReplyDelete
  172. Brandon,

    This is irrelevant to the point; heuristic functions are in themselves neither explanatory nor justifying.

    I am not claiming otherwise. I am just pointing out that it is sometimes the case that one finds it useful to conceptualize what one in fact believes is a false reality. Therefore, even if one believes that no physical causes exist it does not follow that one would have to “tear out causal reasoning”, as I understood you were suggesting while also pointing out how difficult that would be.

    ReplyDelete
  173. Danielos,

    The phrase had a context, the discussion between urban jean and myself, and received its meaning from that context; it cannot be taken out of the argument of which it is apart and treated as if it were not referring to the very specific approach to the question of how to relate scientific theories to causal questions that was being discussed at that point. Your claim, again, is not relevant to any point that was being made, nor in general to the matter that was under discussion at the time, for the reason I already noted.

    ReplyDelete
  174. @Dianelos:

    "And therefore it's not the case that only A-T metaphysics may be called 'classical theism'. Let's have some humility here."

    Humility has little to do with it, and it's not even strictly a matter of A-T metaphysics. You cannot possibly be deliberately asserting, with both a straight face and any serious expectation that a blog full of classical theists won't call you on it, that it is consistent with classical theism for God's creation to include things/events that are not God's creation. And make no mistake, that is what your claim amounts to: an uncaused event is a fortiori not caused by God.

    "[I]n any indeterministic system there are acausal facts. Here's the proof: By definition if a system is deterministic then some state in it can evolve into at least two future states. But then nothing in the system causes it to evolve into one of these potential states rather than the other."

    Prior to roughly the time of Malebranche, causality wasn't about relations between successive "states"; it was about substances doing things and making things happen. On that account of causality, the fact (if it is one) that a human being might make one of two decisions under some specific set of circumstances doesn't in any way mean that he doesn't cause the outcome. So yes, an indeterministic system can still be causal through and through; to say otherwise in the present context is either to equivocate or to beg the question.

    "Surely if God wants to create and give to us a random die, i.e. a die that will produce a random result every time we throw it – it's not like God can't do it."

    To be uncaused in the sense you require entails being uncaused by God; otherwise, as I said earlier, you wouldn't be (as you suppose yourself to be) proposing a counterexample to the PSR.

    So yes, it's very much like God can't do it. At bottom you're saying (at least if you're not equivocating again) that God can make something happen that God didn't make happen, and the contradiction in that proposal is manifest.

    ReplyDelete
  175. And again, if all you really mean to say is that there could in principle be events of which God alone, and nothing in the physical world, is the efficient cause—well, from an A-T point of view that's hardly worth anything more than a brief Of course, and at any rate it's not something you can learn from physics alone, either in general or in any particular case.

    I suggest you sit down and think carefully about just what it is you do mean to say.

    ReplyDelete
  176. Dianelos,

    >> "If an activity is mechanizable, then it is capable of being
    >> mechanized. And if it is capable of being mechanized, then
    >> it is not a something which is, in and of itself, mechanized."

    > Perhaps so, but where did I claim the opposite? I was only
    > arguing that the (presumably designed by humans) machine
    > which produces novel scientific results should not be
    > considered rational.

    You most certainly were not "only arguing that..."

    You were, in fact, attempting to explain why, from your perspective, there isn't anything sad about humans not needing to be truly rational to do science.

    The first paragraph of your attempted explanation makes no mention of machines "produc[ing] novel scientific results".

    Rather, the first paragraph of your attempted explanation talks about chess, and involves an unjustified leap from the existence of chess-playing software to the conclusion that rationality is not needed for humans to play chess.

    My comment was in response to a statement made in that first paragraph.

    ReplyDelete
  177. Scott,

    On that account of causality, the fact (if it is one) that a human being might make one of two decisions under some specific set of circumstances doesn't in any way mean that he doesn't cause the outcome.

    Thanks, that’s a very good point. I stand corrected.

    ReplyDelete
  178. @Dianelos:

    Thank you. That's very gracious of you.

    ReplyDelete
  179. Scott,

    I happen to think that free will is a central issue in the theism versus naturalism debate. It is evident that naturalism is incompatible with free will, but without free will our condition is rendered nonsensical. And it is irrational to believe in a metaphysics that renders our condition nonsensical, especially when there are viable alternatives. Free will only makes sense in a supernaturalistic reality. (When atheists ask for evidence for the supernatural, the immediate answer should be their very own application of free will when they chose to ask this question. They will then probably claim that free will does not really exist and is some kind of illusion – to which one simply retorts that then it’s not the case there is no evidence for the supernatural but only that they refuse to accept the evidence imagining it must be some kind of illusion.)

    Now the error in my “proof” was not to distinguish between mechanical and personal causation. Personal causation through free will appears to be creative, in the sense of introducing something entirely new into reality. But I wonder how A-T metaphysics deals with free will.

    I start with A-T’s principle that any actualized potency is actualized by something already actual. A fact about the human condition is that we often have the potency to choose between A or B. When we act by choosing (say) A, then that potency is actualized. And by the abovementioned principle there is something already actual which actualized that potency. I suppose that “already actual” is something of our nature; we humans have a nature with the power to act in choosing, to make potential choices actual. So far so good.

    But is that act by human nature itself a potency actualized by something actual outside of human nature? I don’t think so, since that would lead to a regression which would remove from us the responsibility for our choice. Is it then correct to say that our nature in respect to free will is “pure act”? I think it was Leibniz who observed that free will makes of us “little gods”.

    I have found a quote by Aquinas which appears to discuss precisely this point:

    Free-will is the cause of its own movement, because by his free-will man moves himself to act. But it does not of necessity belong to liberty that what is free should be the first cause of itself, as neither for one thing to be cause of another need it be the first cause. God, therefore, is the first cause, Who moves causes both natural and voluntary. And just as by moving natural causes He does not prevent their acts being natural, so by moving voluntary causes He does not deprive their actions of being voluntary: but rather is He the cause of this very thing in them; for He operates in each thing according to its own nature.

    So it seems Aquinas disagrees with my analysis – but I don’t understand his reasoning.

    First I don’t understand this: He starts by pointing out that it is not necessarily the case that free will should be the first cause of itself. Agreed. But then he continues “God, therefore, is the first cause”. How does this follow? From the fact that it is not necessarily the case that free will should be the first cause of itself it only follows that perhaps God is that first cause.

    But suppose it is the case that God is the first cause of human free will. But then it is ultimately God who actualizes the potential to choose A rather than B. Aquinas says, arguing from analogy, that this does not render human actions non-voluntary. But I don’t see how the analogy with natural causes is supposed to work. After all there is a huge difference between natural and personal causation (that’s where my proof floundered). Now God is the author of nature. Thus, clearly, by being the first cause of all nature God does not render anything in nature non-natural. But if God is also the first cause of human actual choices (or “actions” as Aquinas puts it), then how can these actions be called voluntary, or free, or entailing personal responsibility?

    ReplyDelete
  180. Hello Brandon, if still reading. Agree entirely about the ubiquity of causal reasoning. Not so sure that "causal reasoning is the only kind of reasoning to the existence of something unknown that has been historically regarded even as reasonably accurate". How does this sit with the hypothetical-deductive reasoning in physics? Regarding causal reasoning in the interpretation of experiments, perhaps we could say something like this: suppose ordinary causal reasoning is in some sense approximate, but good enough to enable us to climb up to the acausally expressed laws of physics. We then kick away the causal ladder and take the postulates and laws of physics as axiomatic. From this foundation we explain how it comes about that the world looks causal to creatures like us and why this is a good approximation. This might be thought circular, but I don't think it is. But, as you say, it's a major project.

    ReplyDelete
  181. Thank you, Scott, for the myJSTOR tip. Not pdfs, or even pdfs of page images, but at least one can read the articles for free. And thanks, DNW, for the pointer to the Collingwood article. Very helpful indeed.

    ReplyDelete
  182. Dianelos,

    ...I don’t understand this: [Aquinas] starts by pointing out that it is not necessarily the case that free will should be the first cause of itself. Agreed. But then he continues “God, therefore, is the first cause”. How does this follow? From the fact that it is not necessarily the case that free will should be the first cause of itself it only follows that perhaps God is that first cause.

    It seems that way, doesn't it? And it seems that way because you are, quite likely, thinking something like this:

    "We know that free will exists, and that it has a first cause. We also know that free will is just one of a gazillion things which exist. If we rule out free will as the first cause of itself, then we are left with a gazillion-1 things, each of which is a candidate for being the first cause of free will. Okay, fine. But how the heck does merely knowing that there are a gazillion-1 candidates for the first cause of free will enable us to know with certainty which particular one of those gazillion-1 candidates is indeed the first cause of free will?"

    The answer, of course, is: it doesn't.

    And Aquinas never said that it does.

    In saying, "God, therefore, is the first cause", Aquinas is not drawing a conclusion from the isolated fact that free-will is not its own first cause; rather, he is returning to a conclusion which had been previously established -- which previously established conclusion is that God "moves causes both natural and voluntary".

    And since God moves causes both natural and voluntary, and free-will is a voluntary cause, it therefore follows that God moves the free-will, i.e., that God is the first cause of free-will.

    But if God is also the first cause of human actual choices (or “actions” as Aquinas puts it), then how can these actions be called voluntary, or free, or entailing personal responsibility?

    The passage you quoted is from A1 of Q83 in the ST, and is a reply to the third of five objections. If the reply to the fifth objection is read, it'll be noticed that not only is man is free to accede to that which would move him, he is also free to not accede, i.e., he is also free to reject an attempt to move him.

    Now, if someone or something endeavors to move me towards a good, and I reject that attempt, on who does it fall that I have not moved towards that good?

    And if someone or something endeavors to move me away from an evil, and I reject that attempt, on who does it fall that I have -- by virtue of that rejection -- elected instead to remain close to or in the evil?

    ReplyDelete
  183. Hi, urban jean,

    How does this sit with the hypothetical-deductive reasoning in physics?

    It's a complicated matter, but to simplify a bit, it doesn't extend beyond already known bounds; that is, it's a process of establishing that a known phenomenon fits into a known pattern or regularity. Or to put it in other words, it isn't a matter of saying that, given a known A, some as-yet-unknown thing should also be there; it only tells you that, given a known A, you already know that B should also be there. This is a much weaker sort of reasoning. To get to new territory with hypothetico-deductive reasoning people usually use analogy, which is not generally regarded as reliable, or causal reasoning, or both.

    The scaffolding approach is interesting, but it looks very much like it requires that causal claims in fact be true -- that they are true only to a degree of approximation doesn't make them any different from almost any other true claims that we make, and it seems that they would have to be true, rather than just heuristics, to be trustworthy enough to the point that we could take the laws we're getting as true themselves rather than just as useful for calculating certain kinds of problems. I doubt, too, that there's any point at which we can actually round the curve -- i.e., given the nature of physics research there will never be a point at which we can take the laws of nature as axiomatic rather than as just being true to at least the best approximations we know on the basis of these experiments that we explain and justify causally. Scaffolding that stays forever is really just an architectural feature. It would be a different matter if a noncausal general theory of evidence could be developed; it's the lack of this that's the real problem, since we're never going to get to a point where evidence is not an essential part of experimental reasoning.

    ReplyDelete
  184. Hi Brandon. The sense in which I think ordinary causal thought is 'approximate' is this. Physics, which I take as my starting point, gives us a very fine-grained description of the world. A macroscopic lump of matter that we can see and feel and heft contains billions of billions of elementary parts. Any lump of matter, but especially living things, undergoes a process of continual exchange of energy and parts with its surroundings. To carve the world into such lumps and to explain what happens in terms of interactions between the lumps is to constrict oneself to an extremely coarse-grained description, which is thus necessarily approximate. For most of us, for most of the time, this is perfectly adequate. Given what we are, our ability to manipulate the world is extremely coarse (in terms of elementary parts) and the communication bandwidth between individuals so low that to be more precise would incur significant costs in time. It just doesn't matter. Unless, say, one is engaged in some extremely delicate scientific or engineering endeavour. To say that striking a match causes a flame leaves so much out: dryness of match, force of strike, roughness of surface, presence of oxygen, and so on. Nevertheless, we have got to the point where a fine-grained description is possible, and we can look back and see why the coarse-grained is sometimes inadequate. And we can take this step, I think, even if we regard the hypothetical fundamental parts as purely instrumental---we can predict how approximate the coarse-grained picture actually is. Someone earlier asked "has physics been reduced to a game where men make the rules rather than trying to discover them? " My reply: physics is a game in which we guess the rules rather than deducing them. It's the best we can do given what we are.

    ReplyDelete
  185. Now, if someone or something endeavors to move me towards a good, and I reject that attempt, on who does it fall that I have not moved towards that good?

    Hold on a second. Is the original claim that God is the first cause only for good actions or is it that God is the first cause of all voluntary actions?

    ReplyDelete
  186. @urban jean:

    "To carve the world into such lumps and to explain what happens in terms of interactions between the lumps is to constrict oneself to an extremely coarse-grained description, which is thus necessarily approximate."

    Where the underlying reality is fine-grained, this is of course true, and no Aristotelian or Thomist would deny it. But a good deal of the world divides naturally into "lumps" (substances), and in those instances higher-level explanations are not only possible but necessary.

    The paradigmatic example is of course a living organism, of which you could conceivably know the states and motions of every subatomic particle and still not know what it was doing in the relevant sense. For example, if I'm swinging my fist into someone's face, nothing in physics is going to tell you whether I'm hitting him deliberately or accidentally, or acting courageously or recklessly.

    If you have a disagreement here, it's going to be about this sort of anti-reductionism, not about the A-T account of causation itself.

    ReplyDelete
  187. Glenn,

    If we rule out free will as the first cause of itself [snip]

    I don’t rule out free will as the first cause of itself. Neither, apparently, does Aquinas. He explicitly says it’s not *necessarily* the case that free will should be the first cause of itself.

    rather, he is returning to a conclusion which had been previously established -- which previously established conclusion is that God "moves causes both natural and voluntary".

    That may be so – can you point out where that conclusion is reached? After all it’s here where Aquinas discusses voluntary causes. Also, if that conclusion is already reached, why does Aquinas state that it is not “necessarily” the case that free will should be the first cause of itself? He should have stated that it’s not the case simpliciter.

    not only is man is free to accede to that which would move him, he is also free to not accede, i.e., he is also free to reject an attempt to move him.

    If I understand you right you are saying this. Suppose in a particular state of affairs one has the potential of actualizing good choice G or evil choice E. If one actually chooses G then God is the first cause, but if one actually chooses E then one is the first cause. Am I understanding you right? For then Step2’s question suggests itself. And, frankly, it’s not clear who is acting here. Something looks amiss.

    I’ve read Aquinas’s respective reply to objection 5 but find it quite opaque (here's a source http://www.newadvent.org/summa/1083.htm ). I don’t think that Ed discusses this specific point in his “Aquinas”. Free will is such a central issue in theism that I’d like very much to understand how A-T metaphysics deals with it. Or perhaps I should say how Aquinas uses A-T metaphysics to deal with it.

    ReplyDelete
  188. Dianelos,

    >> “If we rule out free will as the first cause of itself [snip]”

    > I don’t rule out free will as the first cause of itself. Neither,
    > apparently, does Aquinas. He explicitly says it’s not
    > *necessarily* the case that free will should be the first cause
    > of itself.

    As already mentioned, Aquinas is replying to an objection. And the (shortened) objection is this: "What is 'free is cause of itself'; therefore what is moved by another is not free. But God moves the will; therefore man has not free-will." In order to disarm the objection, it is not necessary to show that it is *impossible* for free will to be the first cause of itself, only to show that it isn't *necessarily* the case that it is.

    (Btw #1: If it is impossible for X to be Y, saying that it is not necessarily the case that X is Y is neither inaccurate nor untrue (it doesn't give a complete picture of the matter, true; but that is another matter, and no blemish here -- Aquinas is not attempting to give a complete picture, but to disarm a very specific, particular objection (and to succeed at that, he need only undercut a single premise of the objection's argument).)

    (Btw #2: If you go back to A3 of Q2 in ST I, then you will find Aquinas explicitly saying, "There is no case known (neither is it, indeed, possible) in which a thing is found to be the efficient cause of itself; for so it would be prior to itself, which is impossible." (My emphasis.))

    >> “rather, he is returning to a conclusion which had been
    >> previously established -- which previously established
    >> conclusion is that God "moves causes both natural and
    >>voluntary".”

    > That may be so – can you point out where that conclusion
    > is reached?

    Try, e.g., A3 of Q2 in ST I. (But don't be overly surprised or disappointed should you fail to locate the exact phrase.)

    > Also, if that conclusion is already reached, why does Aquinas
    > state that it is not “necessarily” the case that free will should
    > be the first cause of itself?

    Already answered.

    >> “not only is man is free to accede to that which would move
    >> him, he is also free to not accede, i.e., he is also free to
    >> reject an attempt to move him.”

    > If I understand you right you are saying this. Suppose in a
    > particular state of affairs one has the potential of actualizing
    > good choice G or evil choice E. If one actually chooses G
    > then God is the first cause, but if one actually chooses E
    > then one is the first cause. Am I understanding you right?

    The statement you're responding to simply says that man is free to accede to or reject an attempted movement, and it says nothing at all about what might be the first cause of the attempted movement.

    Nonetheless...

    > For then Step2’s question suggests itself. And, frankly, it’s
    > not clear who is acting here. Something looks amiss.

    If one is going to think that the correct assignment of the first cause might somehow depend on an eventual outcome, then not only will something look amiss, something will be amiss.

    ReplyDelete
  189. Step2 said...

    Hold on a second. Is the original claim that God is the first cause only for good actions or is it that God is the first cause of all voluntary actions?

    When there is an endeavor to move a person to a good, and he so attenuates, converts, deflects, metastasizes, rejects, translates, etc., it such that he winds up in a state which is (or winds up doing that which is) deprived of good, the first cause is not then shifted from this to that, or reassigned from that to this.

    Nor is the determination of the first cause held in abeyance or in some kind of limbo until such time as an actual outcome, of whatever kind, takes effect.

    This isn't a direct answer to your question; I'm just trying to head things off at the pass.

    ReplyDelete
  190. @Glenn
    It is equally valid to say someone rejects an evil as it is to say they reject a good, both a free choices. Since that is the case, where is the evil cause originating from if there is only a good first cause? From what you've stated so far it seems to be a self-generated first cause for a voluntary act, which is supposedly impossible.

    If one is going to think that the correct assignment of the first cause might somehow depend on an eventual outcome, then not only will something look amiss, something will be amiss.

    Something like teleology.

    ReplyDelete
  191. Step2,

    It is equally valid to say someone rejects an evil as it is to say they reject a good, both a free choices.

    Provided one is talking about different things having been rejected, yes.

    Since that is the case, where is the evil cause originating from if there is only a good first cause?

    The rejection of good results in a privation of good, and the privation of good is what is called evil. If I remember correctly, Aquinas says that evil has no direct cause, only an accidental cause.

    From what you've stated so far it seems to be a self-generated first cause for a voluntary act, which supposedly is impossible.

    Kindly quote what I have said which rightly may be taken as an assertion on my part that a thing may be its own cause, i.e., prior to itself.

    >> If one is going to think that the correct assignment of the
    >> first cause might somehow depend on an eventual outcome,
    >> then not only will something look amiss, something will be
    >> amiss.

    > Something like teleology.

    Uh, yeah, sure. Just like a plane first crashes, then the NTSB conducts an investigation, and, finally, a report is written assigning a cause for the plane having crashed. Clearly, the cause for the plane having crashed didn't exist prior to the NTSB's determination, let alone prior to the plane having crashed. Right.

    ReplyDelete
  192. No, Scott, I really do have a beef with Aristotelian (efficient) causation, as far as I understand it. My alternative account may lead me to a reductive view of living things and you would no doubt regard that as a reductio ad absurdum of my account of causation. But at the moment I'm just trying to home in on what I find deficient not just in Aristotelian causation but present-day common sense causation too.

    ReplyDelete
  193. I'm conscious that we are filling up Ed's blog. People are welcome to continue the conversation here if they wish.

    ReplyDelete
  194. @urban jean:

    Then I don't understand your objection. What you've written is this:

    "[When] one is engaged in some extremely delicate scientific or engineering endeavour[, t]o say that striking a match causes a flame leaves so much out: dryness of match, force of strike, roughness of surface, presence of oxygen, and so on. Nevertheless, we have got to the point where a fine-grained description is possible, and we can look back and see why the coarse-grained is sometimes inadequate."

    Of course. Again, why would you expect an Aristotelian or Thomist to disagree, and how does any of this conflict with Aristotle's fourfold account of causation?

    No A-T theorist would claim that merely knowing the four sorts of cause is sufficient for the purposes of science; metaphysics isn't physics, and nobody has said otherwise. "Striking a match causes flame" is an illustration of a metaphysical principle, not a scientific theory in its own right.

    But the fact that a scientific account is much more detailed doesn't in and of itself mean that the more detailed account doesn't fit the Aristotelian scheme, and I have yet to see any clear and coherent claim that it doesn't. Indeed, the evidence so far suggests that you may simply be insufficiently familiar with that scheme, as indicated by what appeared to be your earlier failure to appreciate the central role that motion (not limited to, but most definitely including, local motion) plays in Aristotelian metaphysics.

    ReplyDelete
  195. @urban jean:

    "Thank you, Scott, for the myJSTOR tip. Not pdfs, or even pdfs of page images, but at least one can read the articles for free."

    You're welcome. Yeah, you can read some articles for free if you read quickly enough; I find that even when I select the option to "keep me logged in," the site still logs me out after ten or fifteen minutes.

    Also, the selection of freely readable articles is far from complete; even when I'm logged in, I can't read, for example, H.A. Prichard's 1943 obituary of H.W.B. Joseph. You read that right: they want $29.00 to let us read a blinkin' obituary.

    ReplyDelete